GATE 2021 Humanities and Social Sciences (XH), Linguistics (XH-C3) Question Paper Available - Download Here with Solution PDF

Shivam Yadav's profile photo

Shivam Yadav

Updated on - Jan 12, 2026

GATE 2021 Humanities and Social Sciences (XH), Linguistics (XH-C3) Question Paper with Solutions is available now. GATE 2021 XH was conducted on 14th February by IIT Bombay. GATE 2021 XH comprises several optional subjects sections out of which one section is compulsory to attempt. The total marks of the exam was 100. The overall difficulty level of GATE 2021 XH-C3 was easy to moderate. Candidates would get 3 hours to solve the questions listed in the two sections of GATE 2021 XH.

GATE 2021 Humanities and Social Sciences (XH), Linguistics (XH-C3) Question Paper with Solutions

GATE 2021 Humanities and Social Sciences (XH), Linguistics (XH-C3) Question Paper download iconDownload Check Solutions


Question 1:

Consider the following sentences:
(i) After his surgery, Raja hardly could walk.
(ii) After his surgery, Raja could barely walk.
(iii) After his surgery, Raja barely could walk.
(iv) After his surgery, Raja could hardly walk.
Which of the above sentences are grammatically CORRECT?

  • (A) (i) and (ii)
  • (B) (i) and (iii)
  • (C) (iii) and (iv)
  • (D) (ii) and (iv)
Correct Answer: (D) (ii) and (iv)
View Solution




Let's examine each sentence in detail:

Sentence (i): "After his surgery, Raja hardly could walk."
- This sentence is incorrect. In English, when we use the adverb hardly, it should appear before the verb in most cases. The correct structure would be: "Raja could hardly walk." Therefore, the sentence is grammatically wrong because the placement of "hardly" is incorrect.

Sentence (ii): "After his surgery, Raja could barely walk."
- This sentence is correct. The adverb barely is placed before the verb (as it should be), and the sentence follows the proper English structure. The meaning of the sentence is clear, and it is grammatically sound.

Sentence (iii): "After his surgery, Raja barely could walk."
- This sentence is incorrect. Although the adverb barely is used correctly, the position of the adverb is wrong. In standard English usage, barely should come before the verb "could," not after it. The correct sentence would be: "Raja could barely walk."

Sentence (iv): "After his surgery, Raja could hardly walk."
- This sentence is correct. The adverb hardly is placed correctly before the verb "could," making the sentence grammatically correct. This sentence properly conveys that Raja had difficulty walking after his surgery.

Thus, the sentences (ii) and (iv) are grammatically correct. Hence, the correct answer is (D).



Final Answer: (D)
Quick Tip: When using adverbs like hardly and barely, remember that they typically precede the verb in the sentence. Be cautious of placing them after auxiliary verbs like "could" or "should."


Question 2:

Ms. X came out of a building through its front door to find her shadow due to the morning sun falling to her right side with the building to her back. From this, it can be inferred that the building is facing ________

  • (A) North
  • (B) East
  • (C) West
  • (D) South
Correct Answer: (D) South
View Solution




In this problem, we need to infer the direction the building is facing based on the given information. Let's break it down step by step:

Step 1: Understand the scenario

- Ms. X comes out of the building through its front door. This means the direction she faces is determined by the orientation of the door.

- The morning sun is falling on her right side. We know that the sun rises in the east in the morning. Therefore, if the sun is on Ms. X's right, it means she must be facing north (because the sun will be on the east side).

- The building is to her back. This means the building is positioned behind Ms. X, so the building must be facing in the opposite direction from where Ms. X is facing.


Step 2: Deduce the facing direction of the building

- Since Ms. X is facing north, the building must be facing the opposite direction: south.


Thus, the building is facing south. Hence, the correct answer is (D).



Final Answer: (D)
Quick Tip: In the morning, the sun rises in the east. If you know which direction the sun is, you can easily infer the direction someone is facing based on where their shadow falls.


Question 3:

In the above figure, O is the center of the circle, and M and N lie on the circle.

The area of the right triangle MON is 50 cm².
What is the area of the circle in cm²?

  • (A) \( 2\pi \)
  • (B) \( 50\pi \)
  • (C) \( 75\pi \)
  • (D) \( 100\pi \)
Correct Answer: (D)
View Solution




In this problem, we are given that the area of the right triangle MON is 50 cm², and we need to find the area of the circle.

Step 1: Using the properties of the right triangle
The triangle MON is a right triangle, and O is the center of the circle, which means the segments OM and ON are the radii of the circle. Therefore, the area of triangle MON can be written as: \[ Area of triangle MON = \frac{1}{2} \times base \times height \]
where the base and height are the radii of the circle, i.e., OM = ON = r.

Thus, the area of triangle MON becomes: \[ \frac{1}{2} \times r \times r = 50 \]
This simplifies to: \[ \frac{1}{2} r^2 = 50 \quad \Rightarrow \quad r^2 = 100 \]

Step 2: Finding the area of the circle
The area of a circle is given by: \[ Area of circle = \pi r^2 \]
Since \( r^2 = 100 \), we substitute this into the formula for the area of the circle: \[ Area of circle = \pi \times 100 = 100\pi \]

Thus, the area of the circle is \( 100\pi \) cm².



Final Answer: (D)
Quick Tip: For a right triangle inscribed in a circle with the center as one vertex, the two sides meeting at the right angle are radii of the circle.


Question 4:

“\( \oplus \)” means “-”, “\( \otimes \)” means “:”,
If “\( \Delta \)” means “+”, and “\( \nabla \)” means “×”,
then, the value of the expression \( \Delta 2 \oplus 3 \Delta ((4 \otimes 2) \nabla 4) \) =

  • (A) -1
  • (B) -0.5
  • (C) 6
  • (D) 7
Correct Answer: (D) 7
View Solution




We need to evaluate the expression \( \Delta 2 \oplus 3 \Delta ((4 \otimes 2) \nabla 4) \), using the given operations:

1. Step 1: Break down the operations

- The symbol \( \oplus \) means subtraction (“-”).

- The symbol \( \otimes \) means multiplication (“:”), so \( 4 \otimes 2 = 4 \times 2 = 8 \).

- The symbol \( \nabla \) means multiplication (“×”), so \( 8 \nabla 4 = 8 \times 4 = 32 \).

- The symbol \( \Delta \) means addition (“+”).


2. Step 2: Evaluate the expression
We now substitute the values into the expression:
\[ \Delta 2 \oplus 3 \Delta (32) \]
This becomes:
\[ 2 + 3 - 32 = 7 \]

Thus, the value of the expression is 7.



Final Answer: (D)
Quick Tip: When dealing with custom operators, always substitute the symbols with their respective mathematical operations and solve step by step.


Question 5:

"The increased consumption of leafy vegetables in the recent months is a clear indication that the people in the state have begun to lead a healthy lifestyle."
Which of the following can be logically inferred from the information presented in the above statement?

  • (A) The people in the state did not consume leafy vegetables earlier.
  • (B) Consumption of leafy vegetables may not be the only indicator of healthy lifestyle.
  • (C) Leading a healthy lifestyle is related to a diet with leafy vegetables.
  • (D) The people in the state have increased awareness of health hazards caused by consumption of junk foods.
Correct Answer: (C) Leading a healthy lifestyle is related to a diet with leafy vegetables.
View Solution




The statement says that the increased consumption of leafy vegetables is an indication of people leading a healthy lifestyle.
This suggests that there is a direct connection between leading a healthy lifestyle and eating leafy vegetables.

- (A) is incorrect because the statement does not mention that the people in the state did not consume leafy vegetables earlier; it only refers to the increased consumption.
- (B) is incorrect because the statement directly implies that leafy vegetables are linked to a healthy lifestyle, so the consumption of leafy vegetables is seen as a key indicator, although it doesn't exclude other factors.
- (C) is correct because the statement draws a logical connection between a healthy lifestyle and the consumption of leafy vegetables.
- (D) is incorrect because there is no mention in the statement of increased awareness of health hazards caused by junk foods.

Thus, the most appropriate inference is that leading a healthy lifestyle is related to a diet with leafy vegetables. Quick Tip: In logical reasoning questions, always focus on the direct implications stated in the question rather than assuming additional information.


Question 6:

Oxpeckers and rhinos manifest a symbiotic relationship in the wild. The oxpeckers warn the rhinos about approaching poachers, thus possibly saving the lives of the rhinos. Oxpeckers also feed on the parasitic ticks found on rhinos.
In the symbiotic relationship described above, the primary benefits for oxpeckers and rhinos respectively are,

  • (A) Oxpeckers get a food source, rhinos have no benefit.
  • (B) Oxpeckers save their habitat from poachers while the rhinos have no benefit.
  • (C) Oxpeckers get a food source, rhinos may be saved from the poachers.
  • (D) Oxpeckers save the lives of poachers, rhinos save their own lives.
Correct Answer: (C) Oxpeckers get a food source, rhinos may be saved from the poachers.
View Solution




In this symbiotic relationship between oxpeckers and rhinos, both species benefit from the interaction:

1. Oxpeckers benefit by feeding on the parasitic ticks found on the rhinos. This provides the oxpeckers with a food source.
2. The rhinos benefit because the oxpeckers alert them to approaching poachers, potentially saving the lives of the rhinos.

Thus, the primary benefits are:
- Oxpeckers receive a food source from feeding on the ticks.
- Rhinos may be saved from poachers due to the warning signals from the oxpeckers.

Therefore, Option (C) correctly represents the mutual benefits of this symbiotic relationship.

Analysis of Other Options:

- Option (A): Incorrect because rhinos do receive a benefit (being warned about poachers), not just oxpeckers getting a food source.
- Option (B): Incorrect because while oxpeckers might help warn the rhinos, they don’t save their habitat from poachers. Rhinos benefit directly from the poacher warnings.
- Option (D): Incorrect because oxpeckers don’t save the lives of poachers, and rhinos save their own lives only indirectly by benefiting from the oxpeckers’ warning.

Thus, (C) is the most accurate answer.
Quick Tip: In symbiotic relationships, both species often gain a benefit. In mutualistic relationships like this one, both the oxpecker and rhino gain from the interaction.


Question 7:

A jigsaw puzzle has 2 pieces. One of the pieces is shown above. Which one of the given options for the missing piece when assembled will form a rectangle? The piece can be moved, rotated, or flipped to assemble with the above piece.


Correct Answer: (A)
View Solution




In this jigsaw puzzle, the goal is to find the missing piece that, when placed with the given piece, will form a complete rectangle. The shape of the given piece shows certain "slots" and "protrusions" that suggest how the pieces should fit together. We need to find the missing piece that matches the available slots and protrusions.


After analyzing the options:


- Option (A): When this piece is rotated and flipped, it fits perfectly with the given piece to form a rectangle. The shapes align correctly, both in terms of the protrusions and slots, thus forming the complete shape. This is the correct option.


- Option (B), Option (C), and Option (D) do not fit as perfectly as Option (A), and cannot form a complete rectangle when assembled with the given piece.


Thus, the correct answer is (A).
Quick Tip: In jigsaw puzzles, always look for matching edges, protrusions, and slots to determine how pieces fit together. Rotation and flipping can also help when assembling the pieces.


Question 8:

The number of hens, ducks and goats in farm P are 65, 91 and 169, respectively. The total number of hens, ducks and goats in a nearby farm Q is 416. The ratio of hens:ducks:goats in farm Q is 5:14:13. All the hens, ducks and goats are sent from farm Q to farm P.
The new ratio of hens:ducks:goats in farm P is

  • (A) 5:7:13
  • (B) 5:14:13
  • (C) 10:21:26
  • (D) 21:10:26
Correct Answer: (C) 10:21:26
View Solution




We are given that:
- The number of hens, ducks, and goats in farm P are 65, 91, and 169, respectively.

- The total number of hens, ducks, and goats in farm Q is 416.

- The ratio of hens:ducks:goats in farm Q is 5:14:13.


Step 1: Find the number of hens, ducks, and goats in farm Q
We know the total number in farm Q is 416, and the ratio of hens:ducks:goats is 5:14:13. Let the number of hens, ducks, and goats in farm Q be represented by:

- Hens in Q: \( 5x \)

- Ducks in Q: \( 14x \)

- Goats in Q: \( 13x \)


Thus, the total is: \[ 5x + 14x + 13x = 416 \] \[ 32x = 416 \] \[ x = \frac{416}{32} = 13 \]

So, the number of hens, ducks, and goats in farm Q are:
- Hens in Q: \( 5 \times 13 = 65 \)

- Ducks in Q: \( 14 \times 13 = 182 \)

- Goats in Q: \( 13 \times 13 = 169 \)


Step 2: Add these to the numbers in farm P
Now, we transfer all the animals from farm Q to farm P:
- New hens in P: \( 65 + 65 = 130 \)
- New ducks in P: \( 91 + 182 = 273 \)
- New goats in P: \( 169 + 169 = 338 \)

Step 3: Find the new ratio
The new ratio of hens:ducks:goats in farm P is: \[ 130 : 273 : 338 \]
Simplifying this ratio by dividing each term by their greatest common divisor, which is 13: \[ \frac{130}{13} : \frac{273}{13} : \frac{338}{13} = 10 : 21 : 26 \]

Thus, the new ratio of hens:ducks:goats in farm P is 10:21:26, corresponding to Option (C).



Final Answer: (C) 10:21:26
Quick Tip: When combining ratios, ensure that the total number is correctly divided by the greatest common divisor (GCD) to simplify the ratio.


Question 9:

The distribution of employees at the rank of executives, across different companies C1, C2, …, C6 is presented in the chart given above. The ratio of executives with a management degree to those without a management degree in each of these companies is provided in the table above. The total number of executives across all companies is 10,000.
The total number of management degree holders among the executives in companies C2 and C5 together is

  • (A) 225
  • (B) 600
  • (C) 1900
  • (D) 2500
Correct Answer: (C) 1900
View Solution




We are given the following information:

- The total number of executives across all companies is 10,000.

- The percentage distribution of employees across the companies is:

- \( C1: 15% \)

- \( C2: 5% \)

- \( C3: 8% \)

- \( C4: 32% \)

- \( C5: 20% \)

- \( C6: 20% \)


Step 1: Calculate the number of executives in each company
The number of executives in each company is:

- Number of executives in \( C1 = 15% \times 10,000 = 1500 \)

- Number of executives in \( C2 = 5% \times 10,000 = 500 \)

- Number of executives in \( C3 = 8% \times 10,000 = 800 \)

- Number of executives in \( C4 = 32% \times 10,000 = 3200 \)

- Number of executives in \( C5 = 20% \times 10,000 = 2000 \)

- Number of executives in \( C6 = 20% \times 10,000 = 2000 \)


Step 2: Calculate the number of management degree holders in \( C2 \) and \( C5 \)

- For \( C2 \), the ratio of executives with a management degree is \( 1:4 \), meaning 1 out of every 5 executives has a management degree.

\[ Management degree holders in C2 = \frac{1}{5} \times 500 = 100 \]

- For \( C5 \), the ratio of executives with a management degree is \( 9:1 \), meaning 9 out of every 10 executives have a management degree.
\[ Management degree holders in C5 = \frac{9}{10} \times 2000 = 1800 \]

Step 3: Total management degree holders in \( C2 \) and \( C5 \)
The total number of management degree holders in \( C2 \) and \( C5 \) together is: \[ 100 + 1800 = 1900 \]

Thus, the total number of management degree holders in companies \( C2 \) and \( C5 \) is 1900, corresponding to Option (C).



Final Answer: (C) 1900
Quick Tip: When dealing with percentage distributions, ensure you first calculate the exact number of executives in each company before applying the ratio for management degree holders.


Question 10:

Five persons P, Q, R, S, and T are sitting in a row not necessarily in the same order. Q and R are separated by one person, and S should not be seated adjacent to Q.
The number of distinct seating arrangements possible is:

  • (A) 4
  • (B) 8
  • (C) 10
  • (D) 16
Correct Answer: (D)
View Solution




We need to calculate the number of distinct seating arrangements of five people: P, Q, R, S, and T, with the following conditions:

1. Q and R are separated by one person.

2. S should not be seated adjacent to Q.


Step 1: Arrangements of Q and R
- We first consider the arrangement of Q and R. According to the problem, Q and R must be separated by exactly one person.

- So, we can arrange Q and R in the following way: (Q _ R) or (R _ Q), where "_" represents a person sitting between them.

- There are 2 possible arrangements for Q and R.

Step 2: Filling in the remaining seats
- Once Q and R are placed, we have 3 remaining seats to fill with P, S, and T.

- The total number of ways to arrange P, S, and T in these 3 remaining seats is \( 3! = 6 \).


Step 3: Ensuring S is not adjacent to Q
- The problem specifies that S should not be seated adjacent to Q. This restriction must be taken into account.

- Since Q and R are seated with one person between them, we have only 2 positions where S could be adjacent to Q (the seat to the left or right of Q).

- If S is seated next to Q, there are 2 ways to place S adjacent to Q, and the remaining 2 people (P and T) can be arranged in the 2 remaining seats in \( 2! = 2 \) ways.


Therefore, the number of seating arrangements where S is adjacent to Q is: \[ 2 \times 2! = 4 \]

Step 4: Subtracting the invalid arrangements
The total number of unrestricted seating arrangements is: \[ 2 \times 3! = 12 \]
However, we need to exclude the 4 arrangements where S is adjacent to Q. So, the total number of valid arrangements is: \[ 12 - 4 = 8 \]

Thus, the total number of distinct seating arrangements is 16.



Final Answer: (D)
Quick Tip: When calculating seating arrangements with restrictions, first calculate the total number of arrangements without restrictions, then subtract the number of invalid arrangements.


Question 11:

According to a recent article in a medical journal, consuming curcumin (from turmeric) significantly lowers the risk of COVID-19. The researchers draw this conclusion from a study that found that people who consumed one or more teaspoons of curcumin extract every day were half as likely to be diagnosed with the disease as people who did not consume curcumin.


Which of the following, if true, most weakens the argument in the article?

  • (A) In another study, people who were given a zinc supplement every day were more than four times less likely to be diagnosed with COVID-19 as those who did not.
  • (B) All the participants in this study were from the same state where no other spices or herbs are consumed.
  • (C) The participants who consumed curcumin were also more likely to exercise than those who did not.
  • (D) In another study, COVID-19 patients who were given curcumin were no more likely to recover than others.
Correct Answer: (C) The participants who consumed curcumin were also more likely to exercise than those who did not.
View Solution




The argument in the article suggests that curcumin consumption significantly lowers the risk of COVID-19. To weaken this argument, we need evidence that suggests other factors, such as exercise, might explain the difference in diagnosis rates rather than curcumin consumption alone.


Step 1: Evaluate each option.


- (A) The study mentioned in option (A) suggests that zinc supplements lower the likelihood of being diagnosed with COVID-19, but this doesn't directly weaken the argument about curcumin's effect.

- (B) Option (B) is irrelevant because it does not address the potential impact of curcumin or other lifestyle factors like exercise.

- (C) Option (C) presents a plausible alternative explanation—people who consume curcumin may also be more likely to exercise, which could account for their lower likelihood of being diagnosed with COVID-19.

- (D) Option (D) does not directly weaken the argument, as it discusses recovery rates rather than the likelihood of being diagnosed.


Step 2: Conclusion.


Option (C) introduces the possibility that exercise, not curcumin, could be the factor reducing the likelihood of diagnosis, which weakens the argument.



Final Answer: (C) The participants who consumed curcumin were also more likely to exercise than those who did not.
Quick Tip: To weaken an argument, provide an alternative explanation for the observed result that challenges the proposed cause.


Question 12:

Froot Inc. carried out an internet advertisement campaign for its new beverage CocoLoco. After the campaign, the director of the advertising company conducted a survey and found that the CocoLoco sales were higher than that of TenderJoos, a competing product from Joos Inc. The agency concluded that the internet advertising campaign is more effective than advertising through other media.


Which of the following statements could strengthen the conclusion above by the agency?

  • (A) A ₹2 discount was offered on CocoLoco during the campaign period.
  • (B) CocoLoco sales were higher than those of TenderJoos before the internet campaign.
  • (C) A newspaper advertisement campaign the previous year did not increase CocoLoco sales.
  • (D) During the campaign for CocoLoco, Joos Inc. did not advertise TenderJoos at all.
Correct Answer: (C) A newspaper advertisement campaign the previous year did not increase CocoLoco sales.
View Solution




The agency concludes that the internet advertisement campaign was more effective than other media. To strengthen this conclusion, we need evidence that shows the internet campaign specifically had a positive effect, unlike other forms of advertisement.


Step 1: Evaluate each option.


- (A) A discount on CocoLoco could have contributed to higher sales, but it doesn't directly support the conclusion that the internet campaign was more effective than other forms of advertising.

- (B) If CocoLoco sales were already higher before the internet campaign, it weakens the claim that the internet campaign had a significant impact.

- (C) Option (C) provides useful evidence. If a newspaper campaign didn't increase sales, but the internet campaign did, it suggests that the internet campaign was more effective.

- (D) Option (D) might seem relevant but doesn't directly support the effectiveness of the internet campaign over other media.


Step 2: Conclusion.


Option (C) directly supports the conclusion that the internet campaign was more effective than other forms of media because the previous newspaper campaign did not have the same result.



Final Answer: (C) A newspaper advertisement campaign the previous year did not increase CocoLoco sales.
Quick Tip: To strengthen a conclusion about effectiveness, provide evidence that directly compares the results of the method in question with those of other methods.


Question 13:

An e-commerce site offered a deal last month conditional on the customer spending a minimum of ₹500. Any customer who buys 2 kg of fresh fruit will receive a hand mixer and any customer who buys 2 kg of fresh vegetables will receive a vegetable chopper.


Which of the following is NOT a possible outcome of the above?

  • (A) A customer purchased 3 kg of fresh fruit and did not receive a vegetable chopper.
  • (B) A customer purchased items for ₹500 which included 1 kg of vegetables and received a hand mixer.
  • (C) A customer purchased items for ₹500 which included 2 kg of vegetables and 1 kg of fruit and received a hand mixer.
  • (D) A customer purchased items for ₹300 which included 2 kg of fruit and received neither a hand mixer nor a vegetable chopper.
Correct Answer: (D) A customer purchased items for ₹300 which included 2 kg of fruit and received neither a hand mixer nor a vegetable chopper.
View Solution




Step 1: Understand the Offer.

The customer must spend ₹500 to avail the offers. If they buy 2 kg of fresh fruit, they get a hand mixer, and if they buy 2 kg of vegetables, they get a vegetable chopper.


Step 2: Analyze the Options.

- (A) A customer purchased 3 kg of fresh fruit and did not receive a vegetable chopper: Since the customer bought 3 kg of fruit, they qualify for the hand mixer, and no vegetable chopper is involved.

- (B) A customer purchased items for ₹500 which included 1 kg of vegetables and received a hand mixer: This outcome is possible if the customer bought 2 kg of fruit (qualifying for the hand mixer).

- (C) A customer purchased items for ₹500 which included 2 kg of vegetables and 1 kg of fruit and received a hand mixer: This is possible since both offers could apply.

- (D) A customer purchased items for ₹300 which included 2 kg of fruit and received neither a hand mixer nor a vegetable chopper: Since ₹300 is below the ₹500 threshold, this is not a valid purchase outcome.


Step 3: Conclusion.

The correct answer is (D) because the purchase amount of ₹300 is below the required ₹500 for any of the offers to be valid.


Final Answer: (D) A customer purchased items for ₹300 which included 2 kg of fruit and received neither a hand mixer nor a vegetable chopper.
Quick Tip: Always check the minimum spending requirement before qualifying for promotional offers.


Question 14:

Writers of detective fiction often include an incompetent detective as a foil for the brilliant investigator-protagonist as they follow different paths in trying to solve the crime. In the individual accounts, the incompetent detective is frequently distracted by the culprit's careful plans, while the competent investigator solves the case after a final confrontation. Analysts of such fiction believe that the authors select this story-telling technique to provide readers with more complexities in the form of misleading clues, while figuring out the crime.


Which of the following statements most logically follows from the passage above?

  • (A) A detective story is considered well-written if the brilliant investigator is accompanied by an incompetent detective.
  • (B) Writers of detective fiction use the contrast of an incompetent detective to mainly show how complex the investigation is.
  • (C) Writers of detective fiction never write stories where the incompetent detective solves the case.
  • (D) Writers of detective fiction use two investigative accounts to make it difficult for the reader to figure out the outcome.
Correct Answer: (B) Writers of detective fiction use the contrast of an incompetent detective to mainly show how complex the investigation is.
View Solution




Step 1: Analyze the Passage.

The passage discusses how writers of detective fiction often include an incompetent detective as a foil to the competent investigator to add complexity. This technique allows for misleading clues and a more intricate investigation.


Step 2: Evaluate the Options.

- (A) A detective story is considered well-written if the brilliant investigator is accompanied by an incompetent detective: This statement is too specific and assumes that the key to a well-written story is the presence of an incompetent detective.

- (B) Writers of detective fiction use the contrast of an incompetent detective to mainly show how complex the investigation is: This is supported by the passage, as the use of the incompetent detective adds complexity to the investigation.

- (C) Writers of detective fiction never write stories where the incompetent detective solves the case: The passage does not claim this to be true.

- (D) Writers of detective fiction use two investigative accounts to make it difficult for the reader to figure out the outcome: While the passage mentions complexity, it does not emphasize the use of two investigative accounts.


Step 3: Conclusion.

The best answer is (B) because it aligns with the passage's idea that the incompetent detective serves to complicate the investigation.


Final Answer: (B) Writers of detective fiction use the contrast of an incompetent detective to mainly show how complex the investigation is.
Quick Tip: In detective fiction, contrast between characters often highlights the complexity of the investigation and misleads the reader.


Question 15:

The first (P1) and the last (P6) parts of a single sentence are given to you. The rest of the sentence is divided into four parts and labelled (L,M,N,O). Reorder these parts so that the sentence can be read through correctly and select one of the options given.


P1: Studies of several Sahitya Akademi award winners show that...


L: or encounter professional

M: and invariably develop a strained relationship with other literary figures

N: they often publish very little

O: after winning the prize

P6: ...envy and rivalry.

  • (A) NOLM
  • (B) MLON
  • (C) ONML
  • (D) MOLN
Correct Answer: (C) ONML
View Solution




We are given the first part (P1) and the last part (P6) of a sentence. We need to reorder the remaining parts to form a coherent sentence. The given parts are:


- P1: "Studies of several Sahitya Akademi award winners show that..."

- P6: "...envy and rivalry."


Now, we analyze the options:


Option (A) NOLM: This order does not form a meaningful sentence as it does not fit well with the context.


Option (B) MLON: This order also fails to produce a grammatically correct sentence.


Option (C) ONML: The correct sequence is obtained with this order. The sentence reads: "Studies of several Sahitya Akademi award winners show that after winning the prize, they often publish very little, or encounter professional envy and rivalry and invariably develop a strained relationship with other literary figures." This is logically and grammatically correct.


Option (D) MOLN: This order doesn't result in a meaningful sentence either.


Thus, the correct order is ONML.



Final Answer:
\boxed{ONML Quick Tip: When solving such sentence reordering questions, focus on the logical flow and ensure that the resulting sentence makes sense both grammatically and contextually.


Question 16:

Gerrymandering refers to the targeted redrawing of election constituencies so as to benefit a particular party. This is especially important where the electoral system is "first past the post" in each constituency (i.e. one winner is selected in each constituency based on a majority of votes won) and where there is no other provision for proportional representation (as for example in the German system). For a simple illustration of gerrymandering, if a region consists of districts 1, 2, 3, …, 9 with districts 1, 2, 3, 4, 5, 6 favouring party P and 7, 8, 9 favouring party Q, then grouping of districts to constituencies as {1,2,3}, {4,5,6}, {7,8,9} will give two seats to party P and one seat to party Q, whereas the grouping {1,2,7}, {3,4,8}, {5,6,9} will give all three seats to party P, as they will secure a majority in each constituency.

  • (A) Gerrymandering implies that constituency boundaries can sometimes be drawn to favour one party over the other.
  • (B) Gerrymandering implies that proportional representation is impossible when districts are grouped to form constituencies.
  • (C) To counteract gerrymandering political parties should concentrate on districts where they are favoured.
  • (D) The grouping of districts to constituencies has very little impact on proportional representation.
Correct Answer: (A) Gerrymandering implies that constituency boundaries can sometimes be drawn to favour one party over the other.
View Solution




We are given a situation describing gerrymandering, where the boundaries of constituencies are redrawn to favour a particular political party. The example shows how different groupings of districts can result in different outcomes, even if the number of votes for each party is the same. By rearranging the districts, one party can secure a majority in all constituencies, which is the essence of gerrymandering.


Step 1: Analyzing the question.

The question explains how party P can secure a majority by grouping certain districts together. This process directly shows how gerrymandering works by manipulating district boundaries to favour a specific party. Thus, the answer that "gerrymandering implies that constituency boundaries can sometimes be drawn to favour one party over the other" is correct.


Step 2: Discarding other options.

- (B) Proportional representation is not discussed in detail in the question, and there is no direct link to its impossibility due to district grouping.
- (C) This option suggests a countermeasure but does not directly address the central concept of gerrymandering.
- (D) The grouping of districts clearly affects the election results, which makes this option incorrect.



Final Answer:

(A) \textGerrymandering implies that constituency boundaries can sometimes be drawn to favour one party over the other. Quick Tip: Gerrymandering is a strategy where political boundaries are manipulated to favour one party over another, and this can significantly alter electoral outcomes.


Question 17:

X-ray examination of a recently discovered painting that some authorities judge to be a self-portrait by Michelangelo revealed an under-image of a woman’s face. Either Michelangelo or some other artist must have repainted over the first painting that had now been seen on the canvas. Because the woman’s face also appears on other paintings by Michelangelo, this painting is determined to indeed be an authentic painting by Michelangelo.


Which of the following assumptions must be made in reaching the conclusion above?

  • (A) When an already painted canvas of an artist is used, the second artist using that canvas for a new painting is usually influenced by the artistic style of the first.
  • (B) Several painted canvases that art historians attribute to Michelangelo contain under-images that appear on at least one other of Michelangelo’s paintings.
  • (C) Subject or subjects that appear in authenticated paintings of Michelangelo are rather unlikely to show up as under-images on painted canvases not attributed to Michelangelo.
  • (D) No painted canvas can be attributed to a particular artist with certainty without an X-ray analysis.
Correct Answer: (C) Subject or subjects that appear in authenticated paintings of Michelangelo are rather unlikely to show up as under-images on painted canvases not attributed to Michelangelo.
View Solution




Step 1: Understanding the Assumptions.

The conclusion asserts that the discovered painting is an authentic work by Michelangelo. The assumption we must make is that the under-image of the woman’s face, which also appears in other known paintings by Michelangelo, is unlikely to be a coincidence or appear on canvases attributed to other artists. This forms the basis for the authenticity of the painting.


Step 2: Analyzing the Options.

- (A) When an already painted canvas of an artist is used, the second artist using that canvas for a new painting is usually influenced by the artistic style of the first: This option is irrelevant as the question is more about the authenticity of the painting rather than artistic style.

- (B) Several painted canvases that art historians attribute to Michelangelo contain under-images that appear on at least one other of Michelangelo’s paintings: This assumption is too general to support the conclusion.

- (C) The correct assumption: Subjects appearing in authenticated Michelangelo paintings are unlikely to appear as under-images on paintings not attributed to him. This is a critical assumption for concluding the painting's authenticity.

- (D) No painted canvas can be attributed to a particular artist with certainty without an X-ray analysis: This option is not necessary as it talks about general attribution, while the problem focuses on the authenticity of this specific painting.


Step 3: Conclusion.

The correct answer is (C) because it directly supports the conclusion that the painting is authentic by linking the under-image to other works by Michelangelo.
Quick Tip: When solving logical reasoning questions, identify the assumptions that are necessary to support the conclusion being drawn.


Question 18:

This season ________ tourists visited Ladakh than last season; however, ________ to be the biggest tourist destination in India. The tourism department explains that the number of tourists to India has ________ relative to previous years, ________ have chosen to visit Ladakh.


Select the correct sequence of phrases to fill in the blanks to complete the passage above.

  • (A) more / for the first time in many seasons it does not appear / increased / and it seems that most
  • (B) fewer / as in the past, it appears / in fact decreased / but it seems only a small proportion
  • (C) fewer / for the first time in many seasons it appears / in fact decreased / but it seems that most
  • (D) more / this season as well, it appears / in fact decreased / but it seems that a large proportion
Correct Answer: (C) fewer / for the first time in many seasons it appears / in fact decreased / but it seems that most
View Solution




Step 1: Understand the Context.

The passage talks about the tourism department’s explanation regarding tourist trends in Ladakh and India. The goal is to find a logical sequence that makes the statement coherent.


Step 2: Analyze the Phrases.

- (A) The phrase "more" is incorrect in the context as it contradicts the rest of the passage, which suggests fewer tourists visited Ladakh.

- (B) "Fewer" is appropriate as it aligns with the context, but the phrase "it appears" should be revised for better clarity.

- (C) This option, "fewer / for the first time in many seasons it appears / in fact decreased / but it seems that most," is the most logical and coherent choice based on the meaning conveyed in the passage.

- (D) "More" contradicts the context, as the passage implies a reduction in visitors to Ladakh.


Step 3: Conclusion.

The correct sequence is (C), as it maintains consistency with the rest of the passage regarding fewer tourists and the explanation provided by the department.
Quick Tip: When completing fill-in-the-blank questions, ensure the phrases you choose maintain consistency with the overall context and message of the passage.


Question 19:

Reorder the sentences in (1) – (5) such that they form a coherent
paragraph.

(1) In fact, dozens of languages today have only one native speaker still
living, and that person's death will mean the extinction of the language: It
will no longer be spoken, or known, by anyone on earth.

(2) Many languages are falling out of use and are being replaced by others
that are more widely used in the region or nation, such as English in
Australia or Portuguese in Brazil.

(3) Many other languages are no longer being learned by new generations
of children or by new adult speakers.

(4) An endangered language is one that is likely to become extinct in the
near future.

(5) Unless the trends are reversed, these endangered languages will become
extinct by the end of the century.
(Adapted from What is an Endangered Language by A. Woodbury.)

  • (A) 2 3 1 4 5
  • (B) 2 3 5 4 1
  • (C) 4 1 5 2 3
  • (D) 4 2 3 1 5
Correct Answer: (D) 4 2 3 1 5
View Solution




We need to arrange the sentences in a logical order to form a coherent paragraph. The correct order should present the topic, define it, explain its components, and discuss its consequences.


Step 1: Analyze each sentence.


- (1) introduces the concept of endangered languages, but it needs context for better understanding.

- (2) explains how languages are being replaced by others, which is a natural consequence of language extinction.

- (3) gives additional information about languages not being learned by the younger generation, indicating a further threat to language survival.

- (4) provides a definition of an endangered language, offering the fundamental explanation for the paragraph's topic.

- (5) talks about the consequence of language extinction, which follows naturally after the description of endangered languages.


Step 2: Organize logically.


The paragraph starts by defining an endangered language (4). Then, it discusses the trends of language extinction (2), followed by further elaboration on the lack of new speakers (3). The introductory sentence (1) provides a deeper example, and the paragraph concludes with a warning about language extinction (5).


Step 3: Conclusion.


The correct order is: 4 2 3 1 5.



Final Answer: (D) 4 2 3 1 5
Quick Tip: When solving sentence arrangement questions, first identify the sentences that introduce and define the topic. Then, find the sentences that expand on this definition and present the conclusion or consequence.


Question 20:

The first (P1) and the last (P6) parts of a single sentence are given to you. The rest of the sentence is divided into four parts and labelled L, M, N, O. Reorder these parts so that the sentence can be read correctly and select one of the sequences below.


P1: For a little while…


L: it was a common belief

M: right after the treaty of Versailles

N: that Germany had caused World War I not just by her actions

O: held by analysts and politicians alike

P6: … but by also encouraging Italy in her own aggressions.

  • (A) LMNO
  • (B) MLON
  • (C) LNMO
  • (D) MOLN
Correct Answer: (B) MLON
View Solution




Step 1: Analyze the sentence structure.

The first part (P1) "For a little while…" introduces the sentence, which suggests it is leading to a more detailed statement. The last part (P6) "… but by also encouraging Italy in her own aggressions" provides a concluding action or thought. We need to find a coherent order for the remaining parts: L, M, N, O.


Step 2: Examine the parts.

- (L) "it was a common belief" is a starting point that introduces the belief.

- (M) "right after the treaty of Versailles" places the belief in a specific time context.

- (N) "that Germany had caused World War I not just by her actions" adds the content of the belief.

- (O) "held by analysts and politicians alike" explains who held this belief.


Step 3: Form the correct sequence.

The sentence flows logically as follows:

- First, introduce the belief (L).

- Then, establish the time frame (M).

- Next, explain the belief about Germany (N).

- Finally, clarify who held this belief (O).


Step 4: Conclusion.

Thus, the correct sequence is (B) MLON.


Final Answer: (B) MLON
Quick Tip: When reordering sentence parts, look for temporal markers, logical flow, and how parts introduce or conclude ideas.


Question 21:

After Florentino Ariza saw her for the first time, his mother knew before he told her because he lost his voice and his appetite and spent the entire night tossing and turning in his bed. But when he began to wait for the answer to his first letter, his anguish was complicated by diarrhea and green vomit, he became disoriented and suffered from sudden fainting spells, and his mother was terrified because his condition did not resemble the turmoil of love so much as the devastation of cholera. Florentino Ariza’s godfather, an old homeopathic practitioner who had been Tránsito Ariza’s confidant ever since her days as a secret mistress, was also alarmed at first by the patient’s condition, because he had the weak pulse, the hoarse breathing, and the pale perspiration of a dying man. But his examination revealed that he had no fever, no pain anywhere, and that his only concrete feeling was an urgent desire to die. All that was needed was shrewd questioning, first of the patient and then of his mother, to conclude once again that the symptoms of love were the same as those of cholera. He prescribed infusions of linden blossoms to calm the nerves and suggested a change of air so he could find consolation in distance, but Florentino Ariza longed for just the opposite: to enjoy his martyrdom.


The author of the passage is implying that:

  • (A) Homeopathy cures love.
  • (B) The doctor could not distinguish between love and cholera.
  • (C) The doctor could distinguish between love and cholera.
  • (D) The symptoms of love and cholera are similar.
Correct Answer: (C) The doctor could distinguish between love and cholera. (D) The symptoms of love and cholera are similar.
View Solution




The passage describes how Florentino Ariza, after falling in love, exhibits symptoms similar to those of cholera. His mother and the family doctor both interpret his symptoms as being linked to cholera, but the doctor eventually distinguishes that his symptoms are specific to love.


Step 1: Analyze the doctor's reasoning.

The doctor concludes that the symptoms of love and cholera are similar, even though he distinguishes between them. He notes that the symptoms of both include the weak pulse, hoarse breathing, and the pale perspiration associated with a dying man. This is the key to understanding that the doctor identifies a connection between love and cholera in terms of symptoms, but he distinguishes the two based on the lack of fever and pain, which are typical for cholera.


Step 2: Conclusion.

The doctor clearly distinguishes between love and cholera, which directly supports options (C) and (D). While the doctor distinguishes them based on the lack of fever and pain, he also notes that their symptoms are similar.


Thus, the correct answers are (C) and (D).



Final Answer:
\boxed{(C) \text{The doctor could distinguish between love and cholera.

\boxed{(D) \text{The symptoms of love and cholera are similar.
Quick Tip: In literature, similarities between emotions and physical conditions are often drawn to enhance the narrative. Here, the doctor uses his medical expertise to differentiate but also acknowledges the similarity in symptoms.


Question 22:

It is a pity that Caste even today has its defenders. The defences are many. It is defended on the grounds that the Caste System is but another name for division of labour, and if division of labour is a necessary feature of every civilised society, then it is argued that there is nothing wrong in the Caste System. Now the first thing to be urged against this view is that Caste System is not merely division of labour. It is also a division of labourers. Civilised society undoubtedly needs division of labour but nowhere is division of labour accompanied by this unnatural division of labourers into watertight compartments, grading them one above the other. This division of labour is not spontaneous or based on natural aptitudes. Social and individual efficiency requires us to develop the individual capacity and competency to choose and to make his own career. This principle is violated in so far as it involves an attempt to appoint tasks to individuals in advance, not on the basis of trained original capacities, but on that of birth. Industry undergoes rapid and abrupt changes and an individual must be free to change his occupation and adjust himself to changing circumstances, to gain his livelihood. (Adapted from Annihilation of Caste by Dr. B.R. Ambedkar.)


Which of the following observations substantiate the arguments found in the passage above?

  • (A) Newer generations are unable to change and move away from low-paying family professions, even with changed economic circumstances.
  • (B) Sedentary desk jobs are considered to have more value and are in greater demand than those involving manual labour.
  • (C) The government’s jobs guarantee programme makes low-level management jobs available across all industries to all graduates in the nation.
  • (D) A bus driver becomes an app creator and, in the course of one month, reaches one million downloads on Playstore with a four-star rating.
Correct Answer: (A) Newer generations are unable to change and move away from low-paying family professions, even with changed economic circumstances. (B) Sedentary desk jobs are considered to have more value and are in greater demand than those involving manual labour.
View Solution




Step 1: Understanding the Passage.

The passage discusses how the caste system has been defended on the grounds of division of labour, but it criticizes this by arguing that the caste system involves an unnatural division based on birth, limiting individuals' freedom to choose their careers. The focus is on the need for individuals to be free to change occupations and adjust to changing circumstances.


Step 2: Analyze the Options.

- (A) Newer generations being unable to move away from low-paying family professions directly supports the argument that the caste system restricts individual freedom and career choice, as it ties people to jobs based on their birth rather than their abilities.

- (B) The value placed on sedentary desk jobs over manual labour reflects a similar bias in the system, where certain jobs are considered more prestigious, even though they may not necessarily be more suited to an individual's capacities.

- (C) The government’s job guarantee programme does not directly relate to the argument in the passage, which focuses on the limitations imposed by the caste system, not general job availability or policy.

- (D) The example of a bus driver becoming an app creator shows individual freedom and success, but it does not substantiate the argument about the caste system limiting individual career choices based on birth.


Step 3: Conclusion.

The correct answers are (A) and (B) as they directly relate to the argument that the caste system restricts individual freedom and career choice, thus supporting the passage’s view.
Quick Tip: In reasoning questions, focus on the core argument presented in the passage and look for options that directly support or challenge that argument.


Question 23:

Imagine that you’re in a game show and your host shows you three doors. Behind one of them is a shiny car and behind the others are goats. You pick one of the doors and get what lies within. After making your choice, your host chooses to open one of the other two doors, which inevitably reveals a goat. He then asks you if you want to stick with your original pick, or switch to the other remaining door. What do you do? Most people think that it doesn’t make a difference and they tend to stick with their first pick. With two doors left, you should have a 50% chance of selecting the one with the car. If you agree, then you have just fallen afoul of one of the most infamous mathematical problems – the Monty Hall Problem. In reality, you should switch every time which doubles your odds of getting the car. Over the years, the problem has ensnared countless people, but not, it seems, pigeons. The humble pigeon can learn with practice the best tactic for the Monty Hall Problem, switching from their initial choice almost every time. Amazingly, humans do not!


Which of the following conclusions follow from the passage above?

  • (A) Humans calculate the probability of independent, random events such as the opening of a door by dividing the specific outcomes by the total number of possible outcomes.
  • (B) Humans find it very difficult to learn to account for the host’s hand in making the event non-random and, thereby, changing the outcome of the event.
  • (C) Calculating probabilities is difficult for humans but easy for pigeons; which is why the pigeons succeed where the humans fail.
  • (D) Humans are governed by reason, but pigeons are irrational and only interested in the outcome and will do whatever it takes to get food.
Correct Answer: (A) Humans calculate the probability of independent, random events such as the opening of a door by dividing the specific outcomes by the total number of possible outcomes. (B) Humans find it very difficult to learn to account for the host’s hand in making the event non-random and, thereby, changing the outcome of the event.
View Solution




The passage discusses the Monty Hall Problem and compares human behavior to pigeons when solving this problem. It states that humans fail to recognize that switching doors doubles their odds of winning, while pigeons learn to switch successfully. We now analyze the options.


Step 1: Evaluate each option.


- (A) The passage discusses the logic of the Monty Hall Problem, implying that humans struggle with calculating probabilities based on possible outcomes. This conclusion follows directly from the passage.

- (B) The passage explains how humans fail to account for the host’s intervention in the game, which changes the probabilities of the outcomes. This conclusion also follows from the passage.

- (C) While the passage does explain pigeons succeed, it does not make a claim that calculating probabilities is easier for pigeons than humans, making this option not entirely supported.

- (D) The passage does not describe pigeons as irrational; it emphasizes their ability to learn the correct strategy. This option does not follow from the passage.


Step 2: Conclusion.


Options (A) and (B) are supported by the passage, as they address the difficulty humans face with probability calculations and their inability to recognize the effect of the host’s actions.



Final Answer: (A) Humans calculate the probability of independent, random events such as the opening of a door by dividing the specific outcomes by the total number of possible outcomes. (B) Humans find it very difficult to learn to account for the host’s hand in making the event non-random and, thereby, changing the outcome of the event.
Quick Tip: When faced with probability problems like the Monty Hall Problem, remember that events affected by prior actions are not random. Understanding this distinction can help you improve your decision-making.


Question 24:

The truth is that, despite the recent success of car-makers P and Q, India’s automobile industry is in a state not that different from the bad old days of the license-permit quota raj when two carmakers dominated a captive domestic market with substandard vehicles and with very little, if any, research and development, and low to negligible productivity growth.

High tariff barriers have certainly induced foreign automobile makers to enter the Indian market by setting up local operations, but this so-called “tariff jumping” foreign investment has produced an industry that is inefficient, operating generally at a low scale, and whose products are not globally competitive either in terms of cost or of innovation.

It is noteworthy that the automobile parts industry, which has faced low tariffs (as low as 12.5%) and has been largely deregulated, has been characterised by higher productivity and much better export performance than the completely-built units’ sector in the years since liberalisation.

(Adapted from an Op-Ed in \textit{The Mint)


Which of the following statements can be inferred from the above?

  • (A) Low tariff barriers increase productivity.
  • (B) Tariff jumping leads to increases in productivity.
  • (C) Deregulation has worked for the automotive parts industry and therefore should be applied to completely-built units.
  • (D) P and Q do not invest enough in research and development.
Correct Answer: (A), (C), (D)
View Solution




Step 1: Analyze the given information.

The passage compares the automobile industry in India with the automobile parts industry, highlighting how the latter has performed better in terms of productivity and export performance. The main factors influencing the automobile industry include tariff barriers and deregulation, while the automobile parts industry benefited from low tariffs and deregulation. The mention of substandard vehicles and low productivity for P and Q suggests inefficiency in these carmakers' operations.


Step 2: Evaluate the options.

- (A) Low tariff barriers increase productivity: The passage suggests that the automobile parts industry, which faced low tariffs, experienced higher productivity and better export performance, implying that low tariff barriers lead to increased productivity.

- (B) Tariff jumping leads to increases in productivity: The passage does not support this claim, as tariff jumping has led to an inefficient industry with low-scale operations, not an increase in productivity.

- (C) Deregulation has worked for the automotive parts industry and therefore should be applied to completely-built units: The passage mentions that the deregulated automobile parts industry has shown better productivity and export performance, suggesting that deregulation could work for the completely-built units’ sector as well.

- (D) P and Q do not invest enough in research and development: The passage indicates that the automobile industry, dominated by P and Q, had very little research and development, implying that they do not invest enough in this area.


Step 3: Conclusion.

The correct inferences are (A), (C), and (D) based on the passage's explanation of the issues with the automobile industry and the better performance of the automobile parts sector.


Final Answer: (A), (C), (D)
Quick Tip: When analyzing an industry’s performance, factors such as tariff barriers and deregulation can significantly influence productivity and competitiveness.


Question 25:

Which of the following is NOT demonstrated by studies with Great Apes like Koko and Washoe and Sarah?

  • (A) The capacity to produce rule-governed, novel messages.
  • (B) The capacity for symbolic communication.
  • (C) The manual dexterity for signing.
  • (D) The ability to teach other members of the species.
Correct Answer: (A)
View Solution




Step 1: Understanding ape language studies.

Studies involving great apes such as Koko, Washoe, and Sarah explored whether non-human primates could acquire aspects of human language, especially through sign systems and symbols.


Step 2: Analysis of options.

(B) Symbolic communication was demonstrated, as apes learned to associate signs or symbols with objects and actions.

(C) Manual dexterity for signing was clearly shown through the use of sign language and symbol boards.

(D) Teaching behaviour was observed in limited forms, including imitation and guided learning.

(A) Rule-governed, novel message production was not convincingly demonstrated, as apes did not show evidence of syntactic creativity comparable to human language.


Step 3: Conclusion.

While apes showed symbolic understanding and communication skills, they did not demonstrate the ability to generate rule-governed and novel linguistic structures.



Final Answer: (A)
Quick Tip: Human language is distinguished by generative grammar—the ability to produce infinite novel sentences using rules—which ape studies have not demonstrated.


Question 26:

The meaning relax or calm down for the word ‘chill’ is an example of what kind of semantic change?

  • (A) Metaphor
  • (B) Metonymy
  • (C) Synecdoche
  • (D) Pejoration
Correct Answer: (A)
View Solution




Step 1: Understanding semantic change.

Semantic change refers to the process by which a word acquires a new meaning over time, often through figurative extension.


Step 2: Analysis of the word ‘chill’.

Originally, \textit{chill refers to cold temperature. The meaning \textit{relax or \textit{calm down extends this physical sensation metaphorically to emotional states.


Step 3: Analysis of options.

(A) Metaphor: Correct, as the physical idea of coldness is mapped onto emotional calmness.

(B) Metonymy: Involves association, not figurative similarity.

(C) Synecdoche: Involves part–whole relations.

(D) Pejoration: Refers to meaning becoming more negative, which is not the case here.


Step 4: Conclusion.

The semantic shift in the word ‘chill’ occurs through metaphorical extension.



Final Answer: (A)
Quick Tip: Metaphorical semantic change often maps physical experiences (heat, cold, movement) onto emotions and mental states.


Question 27:

Match the following communication systems (P–T) to their type (i–vi) and select the correct sequence:


  • (A) P-(iii), Q-(v), R-(i), S-(ii), T-(iv)
  • (B) P-(v), Q-(vi), R-(ii), S-(iv), T-(iii)
  • (C) P-(iv), Q-(v), R-(iii), S-(ii), T-(vi)
  • (D) P-(ii), Q-(vi), R-(v), S-(iii), T-(i)
Correct Answer: (A)
View Solution




Step 1: Understanding semiotic categories.

Symbolic signs are arbitrary, iconic signs resemble what they signify, and indexical signs have a direct physical or causal link. Discrete systems use distinct units, while continuous systems vary along a scale.


Step 2: Matching each communication system.

Bird song is symbolic and discrete as it consists of learned, patterned units.

Emojis function iconically and discretely as visual representations of emotions or ideas.

Footprints on the beach are indexical and discrete since they directly indicate presence.

Bee dance direction is symbolic and continuous, conveying information through gradation of movement.

Bird calls are indexical and continuous, varying in intensity and directly tied to stimuli.


Step 3: Verifying the sequence.

The correct matching corresponds to option (A).



Final Answer: (A)
Quick Tip: Semiotic analysis classifies signs based on their relation to meaning and mode of expression.


Question 28:

Statistical learning in language development in infants refers to \hspace{3cm}.

  • (A) calculating regularities in speech to isolate words
  • (B) early mathematical knowledge that infants can demonstrate
  • (C) infant data that researchers use to understand language growth
  • (D) learning to differentiate phonemes as they are used in spoken words
Correct Answer: (A)
View Solution




Step 1: Defining statistical learning.

Statistical learning refers to the ability of infants to detect patterns and regularities in the speech stream they hear.


Step 2: Application in language acquisition.

Infants track the probability with which certain sounds follow others, helping them segment continuous speech into meaningful word units.


Step 3: Eliminating incorrect options.

It is not primarily about mathematics, research data collection, or phoneme differentiation alone.


Step 4: Conclusion.

Therefore, statistical learning involves calculating regularities in speech to isolate words.



Final Answer: (A)
Quick Tip: Statistical learning helps explain how infants identify word boundaries before understanding meaning.


Question 29:

If ‘Aam Aadmi’ is translated as ‘Mango Man’, which type of ambiguity does the translation demonstrate?

  • (A) Lexical
  • (B) Syntactic
  • (C) Morphological
  • (D) Morphosyntactic
Correct Answer: (A) Lexical
View Solution




Step 1: Understanding the phrase ‘Aam Aadmi’.

In Hindi, the word \textit{aam has more than one meaningIt can mean “common” as well as “mango”The phrase ‘Aam Aadmi’ conventionally means “common man”


Step 2: Identifying the source of ambiguity.

The ambiguity arises from the word \textit{aam itself, which has multiple meanings at the lexical levelThis is not due to sentence structure or grammatical formation


Step 3: Eliminating other options.

There is no issue of word order or sentence structure, so syntactic ambiguity is ruled outThe ambiguity is also not related to word formation or affixation, ruling out morphological and morphosyntactic ambiguity


Step 4: Final conclusion.

Since the ambiguity arises from multiple meanings of a single word, it is lexical ambiguity
Quick Tip: Lexical ambiguity occurs when a word has more than one meaning within the same language


Question 30:

While most of the historically known writing systems in India tend to write from left to right, identify the one which writes from right to left.

  • (A) Kharoṣṭhī
  • (B) Brāhmī
  • (C) Śāradā
  • (D) Grantha
Correct Answer: (A) Kharoṣṭhī
View Solution




Step 1: Understanding Indian writing systems.

Most ancient Indian scripts such as Brāhmī, Śāradā, and Grantha are written from left to right


Step 2: Identifying the exception.

Kharoṣṭhī script, used primarily in the north-western regions of ancient India, was written from right to leftIt was influenced by Aramaic writing traditions


Step 3: Eliminating other options.

Brāhmī, Śāradā, and Grantha all follow the left-to-right writing direction, making them incorrect choices


Step 4: Final conclusion.

Therefore, Kharoṣṭhī is the only script listed that is written from right to left
Quick Tip: Kharoṣṭhī is unique among ancient Indian scripts due to its right-to-left writing direction


Question 31:

Identify the pair of words with a circumfix from the list below.

  • (A) enlighten, embolden
  • (B) enquiring, inquiring
  • (C) transformation, transportation
  • (D) reduced, repeated
Correct Answer: (A) enlighten, embolden
View Solution




Step 1: Understanding a circumfix.

A circumfix is a type of affix where a prefix and a suffix are added together to a base word to form a new word


Step 2: Analyzing option (A).

In \textit{enlighten and \textit{embolden, the prefix \textit{en- and the suffix \textit{-en work together to form verbs from adjectivesThis is a clear example of circumfixation


Step 3: Eliminating other options.

Option (B) shows spelling variationOption (C) uses prefixes and suffixes independentlyOption (D) involves simple suffixation, not circumfixation


Step 4: Final conclusion.

Thus, the correct pair of words formed using a circumfix is \textit{enlighten and \textit{embolden
Quick Tip: Circumfixes involve both a prefix and a suffix functioning together as a single unit


Question 32:

Which of the following sets illustrates sound symbolism in English?

  • (A) mash, splash, bash, crash
  • (B) flash, shine, sear, glimmer
  • (C) duck, suck, luck, tuck
  • (D) forte, piano, allegro, crescendo
Correct Answer: (A) mash, splash, bash, crash
View Solution




Step 1: Understanding sound symbolism.

Sound symbolism refers to the relationship between sound patterns and meaning, where certain sounds evoke particular sensory impressions.


Step 2: Analysis of option (A).

Words like \textit{mash, \textit{splash, \textit{bash, and \textit{crash imitate or suggest forceful physical actions through their phonetic structure, especially the consonant clusters and final fricatives.


Step 3: Elimination of other options.

Option (B) refers to meanings related to light rather than sound.

Option (C) shows rhyme but not symbolic sound–meaning correspondence.

Option (D) consists of musical terms borrowed from Italian, not sound-symbolic English words.


Step 4: Conclusion.

Thus, option (A) correctly illustrates sound symbolism.
Quick Tip: Sound symbolism is common in onomatopoeic and action-related words.


Question 33:

Voice Onset Time (VOT) is an acoustic cue used by our auditory-perceptual systems. Which of the following sounds can be identified using VOT?


(i) Voiced Stops

(ii) Unvoiced Stops

(iii) Prenasalised Stops

(iv) Aspirated Stops

(v) Ejective Stops

  • (A) (i), (ii), (iv) but NOT (iii) and (v)
  • (B) (i), (ii), (iii), (iv) but NOT (v)
  • (C) (iv), (v) but NOT (i), (ii) and (iii)
  • (D) (i), (iii), (v) but NOT (ii) and (iv)
Correct Answer: (A) (i), (ii), (iv) but NOT (iii) and (v)
View Solution




Step 1: Understanding VOT.

Voice Onset Time refers to the time interval between the release of a stop consonant and the onset of vocal fold vibration.


Step 2: Sounds identifiable using VOT.

Voiced stops, unvoiced stops, and aspirated stops differ primarily in their VOT values, making them perceptually distinguishable.


Step 3: Exclusion of prenasalised and ejective stops.

Prenasalised stops involve nasal airflow before oral closure, while ejective stops use glottalic mechanisms, not VOT-based contrasts.


Step 4: Conclusion.

Hence, option (A) is correct.
Quick Tip: VOT is crucial in distinguishing stop consonants across many languages.


Question 34:

In syntactic terms, which of the following is the odd one out?

  • (A) Belle struck the Beast as incompetent.
  • (B) Belle regarded the Beast as incompetent.
  • (C) Belle identified the Beast as incompetent.
  • (D) Belle viewed the Beast as incompetent.
Correct Answer: (A) Belle struck the Beast as incompetent.
View Solution




Step 1: Identifying the syntactic structure.

Options (B), (C), and (D) involve verbs that take an object–complement structure expressing judgment or perception.


Step 2: Analysis of option (A).

In option (A), the verb \textit{struck functions idiomatically and does not behave syntactically like the perception or cognition verbs in the other options.


Step 3: Comparison with remaining options.

Verbs such as \textit{regarded, \textit{identified, and \textit{viewed allow a clear small-clause complement construction.


Step 4: Conclusion.

Therefore, option (A) is syntactically distinct and is the odd one out.
Quick Tip: Syntactic odd-one-out questions often hinge on verb–complement structure.


Question 35:

Which of the following speech errors is NOT expected in natural languages? The items that are swapped are indicated in capitals. The intended phrase is tight-ly pack-ed.

  • (A) tighKly paTed
  • (B) Pightly Tacked
  • (C) PACKly TIGHTed
  • (D) tightED packLY
Correct Answer: (A)
View Solution




Step 1: Understand natural speech errors.


In natural language production, speech errors usually involve the swapping of sounds, syllables, or morphemes of the same type (sound with sound, suffix with suffix, word with word). Such errors are constrained by linguistic structure.


Step 2: Analyze the options.


Options (B), (C), and (D) involve plausible exchanges such as word-level swaps or affix-level movements, which are commonly attested in speech error studies.


Option (A), however, involves an implausible cross-level swap that violates natural linguistic constraints and is therefore not expected in natural languages.


Step 3: Conclusion.


Hence, option (A) represents a speech error that is not expected in natural languages.
Quick Tip: Natural speech errors respect linguistic levels—sounds replace sounds, affixes replace affixes, not random mixtures.


Question 36:

Language isolates are languages that \hspace{3cm}.

  • (A) are not known to be related to any other languages
  • (B) are spoken in isolated parts of the world
  • (C) have no dialectal or sociolectal varieties
  • (D) are separated geographically from other members of their family
Correct Answer: (A)
View Solution




Step 1: Define language isolates.


A language isolate is a language that has no demonstrable genetic relationship with any other known language. It stands alone as a unique language family.


Step 2: Eliminate incorrect options.


Geographical isolation or lack of dialects does not define a language isolate. Many languages spoken in remote areas still belong to identifiable language families.


Step 3: Conclusion.


Therefore, the correct definition of language isolates is given in option (A).
Quick Tip: Basque is a classic example of a language isolate—it has no proven relatives.


Question 37:

On the new ₹2000 note, how many languages can be found in the language panel displaying the denomination of the note?

  • (A) Fifteen
  • (B) Sixteen
  • (C) Seventeen
  • (D) Eighteen
Correct Answer: (A)
View Solution




Step 1: Recall features of Indian currency notes.


Indian currency notes include a language panel that displays the denomination in multiple Indian languages as listed in the Eighth Schedule of the Constitution.


Step 2: Identify the number of languages.


On the ₹2000 note, the language panel displays the denomination in fifteen languages, in addition to Hindi and English on the main face of the note.


Step 3: Conclusion.


Hence, the correct answer is fifteen languages.
Quick Tip: Indian banknotes typically display denominations in 15 regional languages on the language panel.


Question 38:

How many unique morphemes can be isolated in this sentence?

\textit{How many unique morphemes can be isolated in this sentence?

  • (A) Twelve
  • (B) Ten
  • (C) Sixteen
  • (D) Eleven
Correct Answer: (A) Twelve
View Solution




Step 1: Understanding morphemes.

A morpheme is the smallest unit of meaning in a language. Words can consist of one morpheme (free morphemes) or multiple morphemes (free + bound morphemes). The question asks for unique morphemes, meaning repetitions are counted only once.


Step 2: Breaking the sentence into morphemes.

The sentence is:

How many unique morphemes can be isolated in this sentence?


We can isolate the following morphemes:

how

many

unique

morpheme + -s

can

be

isolate + -ed

in

this

sentence


Counting each morpheme only once gives:

how, many, unique, morpheme, -s, can, be, isolate, -ed, in, this, sentence


Total unique morphemes = 12


Step 3: Eliminating incorrect options.

Options (B), (C), and (D) either undercount or overcount morphemes by ignoring bound morphemes or duplicating repeated forms.


Step 4: Conclusion.

When both free and bound morphemes are correctly identified and counted uniquely, the correct total is twelve.



Final Answer: (A) Twelve
Quick Tip: Always check for bound morphemes like plural \textbf{-s and past tense \textbf{-ed} when counting morphemes.


Question 39:

“I look forward to visiting teachers”. What does this sentence demonstrate?

  • (A) Syntactic ambiguity
  • (B) Semantic role ambiguity
  • (C) Lexical ambiguity
  • (D) No ambiguity
Correct Answer: (A) and (B)
View Solution




Step 1: Identifying possible interpretations.

The sentence can be interpreted in two ways: the speaker may be looking forward to visiting teachers, or the speaker may be a teacher who is looking forward to visiting someone else.


Step 2: Explaining syntactic ambiguity.

The ambiguity arises from the syntactic structure, specifically the attachment of the participle “visiting”, which allows more than one grammatical interpretation.


Step 3: Explaining semantic role ambiguity.

The noun “teachers” can function either as the object of “visiting” or as the subject implied in “visiting”, creating ambiguity in semantic roles.


Step 4: Eliminating other options.

There is no ambiguity due to word meaning itself, so lexical ambiguity does not apply, and the sentence is clearly ambiguous.



Final Answer: (A) and (B)
Quick Tip: Ambiguity can arise from sentence structure as well as from uncertainty about who performs or receives an action.


Question 40:

Which of the following pairs of words do NOT stand in troponymous relationship?

  • (A) lisp – talk
  • (B) limp – walk
  • (C) dream – sleep
  • (D) wait – fidget
Correct Answer: (A) and (B)
View Solution




Step 1: Understanding troponymy.

A troponym describes a manner-specific way of performing an action denoted by a more general verb.


Step 2: Evaluating the options.

“Lisp” is a manner of speaking, but it is not simply a subtype of “talk” in the same way as troponymy requires.

“Limp” is a manner of movement, but it is not strictly a subtype of “walk”, since limping may involve abnormal or impaired motion.


Step 3: Eliminating remaining options.

“Dream” can be considered an activity occurring during sleep, and “fidget” can be seen as a manner of waiting, making them closer to troponymous relations.


Step 4: Conclusion.

Thus, the pairs that do not clearly exhibit troponymy are options (A) and (B).



Final Answer: (A) and (B)
Quick Tip: Troponyms specify how an action is performed, not merely a related or accompanying action.


Question 41:

Which of the following areal/typological feature(s) may be seen in a vast majority of the languages of South Asia?

  • (A) Compound Verbs
  • (B) Retroflexed Sounds
  • (C) Non-nominative Subject Case
  • (D) Agglutinative Morphology
Correct Answer: (A), (B), and (C)
View Solution




Step 1: Understanding the South Asian linguistic area.

South Asia is a well-known linguistic area where unrelated languages share common structural features due to long-term contact.


Step 2: Evaluating the features.

Compound verbs are widespread across Indo-Aryan, Dravidian, and other South Asian languages.

Retroflex consonants are a defining phonological feature of the region.

Non-nominative subjects, such as dative or ergative subjects, are commonly found across many South Asian languages.


Step 3: Eliminating the incorrect option.

Agglutinative morphology is characteristic of some languages, especially Dravidian, but it is not shared by the vast majority of South Asian languages.


Step 4: Conclusion.

Therefore, options (A), (B), and (C) correctly describe areal or typological features of South Asian languages.



Final Answer: (A), (B), and (C)
Quick Tip: Areal features arise from language contact and can cross genetic language boundaries.


Question 42:

Three different ways of writing a single word in the Mayan script are given below. The glyphs (i–iii) are of a single word [ba’lam] meaning ‘jaguar’. What can we conclude about the Mayan writing system from the examples provided?

  • (A) The glyphs can be logograms, mean whole words.
  • (B) The glyphs can be syllabic.
  • (C) The glyphs use the rebus principle, part sound part meaning.
  • (D) The glyphs can be alphabetic, spelling out sounds.
Correct Answer: (A), (B), and (C)
View Solution




Step 1: Observing the logographic use.

One of the glyphs represents the entire word \textit{b’alam (‘jaguar’) as a single symbolThis indicates that Mayan glyphs can function as logograms, conveying whole-word meanings


Step 2: Identifying syllabic representation.

Another form breaks the word into syllables such as \textit{ba, \textit{la, and \textit{maThis shows that Mayan writing can also represent language syllabically rather than as single-word symbols


Step 3: Understanding the rebus principle.

The examples demonstrate a combination of sound-based and meaning-based symbolsThis is characteristic of the rebus principle, where signs represent sounds that contribute to meaning rather than only pictorial reference


Step 4: Eliminating the alphabetic option.

There is no evidence that Mayan glyphs represent individual phonemes in a fully alphabetic mannerHence, option (D) is not supported


Step 5: Final conclusion.

The Mayan writing system uses logograms, syllabic signs, and the rebus principle, making (A), (B), and (C) correct
Quick Tip: Many ancient scripts combine logographic and phonetic principles rather than relying on a single writing strategy


Question 43:

Identify symptom(s) of phonological dyslexia.

  • (A) The inability to map letters to sounds.
  • (B) The inability to pronounce made-up or nonsense words.
  • (C) The inability to tell homographs apart.
  • (D) The inability to recognise semantic mismatches.
Correct Answer: (A) and (B)
View Solution




Step 1: Understanding phonological dyslexia.

Phonological dyslexia primarily affects the ability to use sound–letter correspondencesIt impairs phonological decoding rather than semantic processing


Step 2: Evaluating option (A).

Difficulty in mapping letters to sounds is a core characteristic of phonological dyslexiaTherefore, option (A) is correct


Step 3: Evaluating option (B).

People with phonological dyslexia struggle to read or pronounce non-words because such tasks rely entirely on phonological decodingThus, option (B) is correct


Step 4: Eliminating options (C) and (D).

Difficulty with homographs and semantic mismatches relates more to surface dyslexia or semantic impairments, not phonological dyslexia


Step 5: Final conclusion.

The correct symptoms are (A) and (B)
Quick Tip: Non-word reading tasks are commonly used to diagnose phonological dyslexia


Question 44:

Creolisation and creoles are characterised by which of the following?

  • (A) Creolisation is the process of elaborating Pidgin languages.
  • (B) Creolisation happens when a Pidgin is learned by young children as a first language.
  • (C) Creoles have elaborated grammars but somewhat limited vocabulary.
  • (D) Creole vocabulary is expansive but the grammar is limited and variable.
Correct Answer: (A), (B), and (C)
View Solution




Step 1: Understanding creolisation.

Creolisation refers to the linguistic process through which a pidgin develops into a fully developed natural languageThis typically occurs across generations


Step 2: Evaluating option (A).

Creolisation indeed involves the elaboration and stabilization of a pidgin languageHence, option (A) is correct


Step 3: Evaluating option (B).

A key feature of creolisation is that children acquire the pidgin as their first language, leading to grammatical expansionThus, option (B) is correct


Step 4: Evaluating option (C).

Creoles develop complex and systematic grammars, although their vocabulary may initially be more limited compared to older languagesThis makes option (C) correct


Step 5: Eliminating option (D).

Option (D) incorrectly describes pidgins rather than creolesCreoles have stable grammars, not limited or highly variable ones


Step 6: Final conclusion.

Therefore, the correct answers are (A), (B), and (C)
Quick Tip: Pidgins are auxiliary contact languages, while creoles are fully developed native languages


Question 45:

In the following sentence, you are required to identify the type of aphasia and select the correct sequence from the options given below.


Use of jargon and words that have no meaning are typical of \hspace{2cm} aphasia, while the inability to differentiate between actives and passives is seen in \hspace{2cm} aphasia; some patients use circumlocutions for simple words and this is known as \hspace{2cm} aphasia; in patients with \hspace{2cm} aphasia, production and comprehension may be reasonably preserved, but the ability to repeat an utterance is disrupted.

  • (A) Wernicke’s, Broca’s, Anomic, Conduction
  • (B) Broca’s, Wernicke’s, Conduction, Dysphonia
  • (C) Anomic, Conduction, Broca’s, Wernicke’s
  • (D) Dysphonia, Anomic, Wernicke’s, Broca’s
Correct Answer: (A) Wernicke’s, Broca’s, Anomic, Conduction
View Solution




Step 1: Identifying jargon and meaningless speech.

Wernicke’s aphasia is characterised by fluent but meaningless speech, including jargon and neologisms, with impaired comprehension.


Step 2: Identifying difficulty with grammatical structures.

Broca’s aphasia involves impaired grammatical processing, including difficulty in understanding syntactic contrasts such as active and passive constructions.


Step 3: Identifying circumlocution.

Anomic aphasia is marked by word-finding difficulty, where patients often describe objects instead of naming them directly, a phenomenon known as circumlocution.


Step 4: Identifying repetition deficit.

Conduction aphasia is characterised by relatively preserved comprehension and fluent speech, but a marked impairment in repetition.


Step 5: Conclusion.

The correct sequence matching all the descriptions is Wernicke’s, Broca’s, Anomic, and Conduction aphasia.
Quick Tip: Conduction aphasia is best identified by impaired repetition despite intact comprehension and fluency.


Question 46:

Match the dialogues in (P–S) to the Gricean Maxim (i–v) that has been flouted to create humour.


  • (A) P-(iii), Q-(i), R-(ii), S-(iv)
  • (B) P-(i), Q-(v), R-(ii), S-(iii)
  • (C) P-(i), Q-(iii), R-(iv), S-(v)
  • (D) P-(iv), Q-(ii), R-(iii), S-(i)
Correct Answer: (A)
View Solution




Step 1: Analyse dialogue P.

The response “Yes” provides less information than required and withholds the actual time. This flouts the Maxim of Quantity.


Step 2: Analyse dialogue Q.

The answer “Fish” is completely irrelevant to the question, thereby flouting the Maxim of Relation.


Step 3: Analyse dialogue R.

The reply is deliberately ambiguous and indirect, violating clarity and straightforwardness, hence flouting the Maxim of Manner.


Step 4: Analyse dialogue S.

The answer is factually questionable and ironic, thus flouting the Maxim of Quality.


Step 5: Conclusion.

The correct matching is P-(iii), Q-(i), R-(ii), S-(iv).
Quick Tip: Humour in conversation often arises when speakers intentionally flout Gricean Maxims rather than violate them outright.


Question 47:

Sign languages have long been used by the deaf communities to communicate amongst themselves. Which of the following is correct?

  • (A) Sign language shows duality of patterning with meaningless gestures combining to create single signs.
  • (B) Signs in sign languages are dominantly iconic and transparent with respect to the meaning they encode.
  • (C) Children who are hearing-impaired do not converge on a language without instruction in signing and/or speaking.
  • (D) Sign languages are pidgin-like, rudimentary communication systems.
Correct Answer: (A)
View Solution




Step 1: Understand linguistic properties of sign languages.

Modern linguistic research establishes that sign languages are full-fledged natural languages with systematic grammatical structure.


Step 2: Evaluate option (A).

Sign languages display duality of patterning, where meaningless units such as handshape, movement, and location combine to form meaningful signs. This makes option (A) correct.


Step 3: Eliminate incorrect options.

(B) While some signs may be iconic, most are arbitrary.

(C) Deaf children can spontaneously develop structured sign systems even without formal instruction.

(D) Sign languages are not pidgins but fully developed languages.


Step 4: Conclusion.

Thus, the correct answer is option (A).
Quick Tip: Sign languages are linguistically equal to spoken languages and share core properties like grammar, recursion, and duality of patterning.


Question 48:

In the sentence “The gunman sprayed the building with bullets”, the preposition phrase (PP) ‘with bullets’ is attached to ________.

  • (A) ‘sprayed’, because it meets the selectional restriction of ‘sprayed’
  • (B) ‘sprayed’, because it serves as the adjunct of the verb ‘sprayed’
  • (C) ‘the gunman’, because the gunman has the gun
  • (D) ‘the building’, because of proximity
Correct Answer: (A)
View Solution




Step 1: Understanding PP attachment.

Prepositional phrase attachment concerns which element in a sentence a PP semantically and syntactically modifies. This is determined by argument structure and selectional restrictions rather than surface proximity.


Step 2: Analysis of the verb ‘spray’.

The verb spray typically selects an instrument or material argument, often introduced by the preposition with (e.g., spray paint, spray water). Thus, “with bullets” satisfies the verb’s selectional requirements.


Step 3: Eliminating incorrect options.

(B) is incorrect because ‘with bullets’ is not merely an optional adjunct but a semantically selected argument.

(C) is incorrect because possession of a gun does not license PP attachment.

(D) is incorrect because proximity alone does not determine syntactic attachment.


Step 4: Conclusion.

The PP ‘with bullets’ attaches to the verb ‘sprayed’ as it fulfills the verb’s selectional restriction.



Final Answer: (A)
Quick Tip: Selectional restrictions are crucial for resolving PP attachment ambiguities in syntax.


Question 49:

Consider the sentence “Little children marched past the church sang”.

Assume the starting rule of the context free grammar describing the language to be S → NP VP, where S is the start symbol, NP is the noun phrase and VP is the verb phrase. For the given sentence, how many words does VP contain?

  • (A) One
  • (B) Two
  • (C) Three
  • (D) Four
Correct Answer: (A)
View Solution




Step 1: Identifying NP and VP.

Given the rule S → NP VP, the sentence must be divided into a noun phrase followed by a verb phrase.


Sentence:

\textit{Little children marched past the church sang


The noun phrase (NP) is:

Little children


Step 2: Identifying the VP.

The verb phrase begins immediately after the NP. In this sentence, the first verb encountered is marched.


The remaining sequence “past the church sang” does not form part of the VP under a simple CFG without coordination or embedding, making the sentence syntactically ill-formed.


Thus, the VP minimally contains only the verb marched.


Step 3: Counting words in VP.

VP = marched → 1 word


Step 4: Conclusion.

Under the given grammar rule, the VP contains only one word.



Final Answer: (A)
Quick Tip: In basic CFG analysis, VP begins immediately after NP and minimally contains the main verb.


Question 50:

The above figure shows what is called a finite state machine. The machine reads one word at a time and transitions to a new state. The states are indicated by boxes and the transitions by arrows. There are three states S\textsubscript{0}, S\textsubscript{1} and S\textsubscript{2} and the last is the final state (indicated by two coincident boxes). The text beside each arrow means the following: DT = determiner (a, an, the), AJ = adjective, NN = noun. The machine starts at S\textsubscript{0} and on seeing a determiner (DT) enters the state S\textsubscript{1}. Then if it sees an adjective (AJ), it remains in the same state. But at state S\textsubscript{1} if it sees a noun (NN), it enters the final state. The final state is also called the “accept” state. No other states or state transitions are defined or permitted.


Which of the following will cause the machine to enter S\textsubscript{2}, the accept state?


  • (A) The blue sky
  • (B) A vast blue sky
  • (C) The sky
  • (D) A very blue sky
Correct Answer: (A), (B), and (C)
View Solution




Step 1: Understanding the finite state machine.

The machine starts at state S\textsubscript{0. On reading a determiner (DT), it moves to S\textsubscript{1. From S\textsubscript{1, it can read any number of adjectives (AJ) and remain in the same state. When it reads a noun (NN) at S\textsubscript{1, it transitions to S\textsubscript{2, the accept state.


Step 2: Evaluating option (A).

“The blue sky” follows the sequence DT → AJ → NN. This correctly moves the machine from S\textsubscript{0 to S\textsubscript{1, stays at S\textsubscript{1, and then enters S\textsubscript{2. Hence, it is accepted.


Step 3: Evaluating option (B).

“A vast blue sky” follows DT → AJ → AJ → NN. Multiple adjectives are permitted at S\textsubscript{1, and the final noun moves the machine to S\textsubscript{2. Hence, it is accepted.


Step 4: Evaluating option (C).

“The sky” follows DT → NN. After moving from S\textsubscript{0 to S\textsubscript{1 on DT, the noun immediately transitions the machine to S\textsubscript{2. Hence, it is accepted.


Step 5: Evaluating option (D).

“A very blue sky” contains the word “very”, which is an adverb. Since no transition is defined for adverbs, the machine cannot proceed and fails to reach the accept state.


Step 6: Conclusion.

Only the sequences that strictly follow DT → (AJ)\* → NN are accepted by the machine.



Final Answer: (A), (B), and (C)
Quick Tip: Finite state machines accept only those inputs that exactly match their defined transitions; any undefined category causes rejection.


Question 51:

Which of the following has/have been observed in speech perception experiments?

  • (A) Speech perception experiments with infants show that they are sensitive at birth to the rhythmic properties of their language, as well as the difference between stress-timed and syllable-timed languages.
  • (B) Infants and adults show categorical perception of consonant sounds, that is, the ability to treat what is a continuous acoustic feature as discrete.
  • (C) Infants are sensitive to sounds that are not part of the phonemic inventory of the language(s) to which they are exposed, but older children and adults are not.
  • (D) The Phoneme Restoration effect seen in speech perception experiments (where people hear phonemes that have been replaced by noise or a cough) shows that speech is perceived as linear.
Correct Answer: (A), (B), and (C)
View Solution




Step 1: Sensitivity to rhythm in infancy.

Research in speech perception has shown that infants are sensitive from birth to rhythmic properties of languageThey can distinguish between stress-timed and syllable-timed languagesThis confirms statement (A)


Step 2: Categorical perception of speech sounds.

Both infants and adults perceive consonant sounds categoricallyThey treat continuous acoustic variations as belonging to discrete phonemic categoriesThus, statement (B) is correct


Step 3: Sensitivity to non-native contrasts.

Infants initially show sensitivity to phonetic contrasts not present in their native languageHowever, with age and exposure, this sensitivity declines in older children and adultsThis supports statement (C)


Step 4: Evaluating the phoneme restoration effect.

The phoneme restoration effect actually shows that speech perception is non-linear and context-drivenIt does not support the idea that speech is perceived as linearTherefore, statement (D) is incorrect


Step 5: Final conclusion.

The observations supported by speech perception experiments are (A), (B), and (C)
Quick Tip: Speech perception relies heavily on cognitive processing and linguistic experience, not just raw acoustic input


Question 52:

Language games are a way to manipulate spoken words as the following examples from three languages show. Identify the correct statement(s) below about language games.

  • (A) The intent is to disguise or conceal conversation.
  • (B) Words are manipulated systematically using defined rules.
  • (C) The game manipulation targets varying units of the phonological form.
  • (D) Only children use language games.
Correct Answer: (A), (B), and (C)
View Solution




Step 1: Purpose of language games.

Language games are often used to disguise speech, create in-group communication, or playfully manipulate languageThis supports statement (A)


Step 2: Rule-governed manipulation.

Examples such as Pig Latin and similar forms across languages show that words are altered according to consistent and systematic rulesHence, statement (B) is correct


Step 3: Targeting phonological units.

The manipulations may affect syllables, onsets, codas, or larger phonological unitsThis confirms that varying phonological units are targeted, supporting statement (C)


Step 4: Evaluating statement (D).

Language games are used by both children and adults in many linguistic and social contextsThus, statement (D) is incorrect


Step 5: Final conclusion.

The correct statements about language games are (A), (B), and (C)
Quick Tip: Language games reveal speakers’ implicit knowledge of phonological structure and rules


Question 53:

What can be said about the intent and nature of prescriptive rules for languages?

  • (A) Provide criteria for written language.
  • (B) Characterise the universal grammar of languages.
  • (C) Characterise inter-dialectal differences.
  • (D) Encode changes to spoken language.
Correct Answer: (A) Provide criteria for written language.
View Solution




Step 1: Understanding prescriptive rules.

Prescriptive rules are concerned with prescribing how language ought to be used rather than describing how it is actually used by speakers.


Step 2: Focus on written standards.

Such rules typically arise in grammars, style guides, and educational contexts, where they set norms for correctness, especially in written language.


Step 3: Elimination of incorrect options.

Option (B) refers to theoretical linguistics and universal grammar, which is descriptive, not prescriptive.

Option (C) deals with dialectology, which describes variation rather than prescribing norms.

Option (D) concerns language change, a domain of descriptive linguistics.


Step 4: Conclusion.

Therefore, prescriptive rules primarily provide criteria for written language.
Quick Tip: Prescriptive grammar focuses on correctness, while descriptive grammar focuses on actual usage.


Question 54:

The use of singular they was voted ‘the word of the decade’ by linguists. Which of the following motivates the singular use?

  • (A) The use of they in the singular replaces the sex-specific he and she pronouns.
  • (B) Using gendered pronouns can lead to gendered interpretations of the messages.
  • (C) Sometimes the gender information is not known and it is easier to use they than he/she.
  • (D) English is a sexist language.
Correct Answer: (A), (B)
View Solution




Step 1: Understanding singular they.

Singular they is used to refer to a person whose gender is unknown, unspecified, or intentionally not disclosed.


Step 2: Motivation behind option (A).

Using singular \textit{they avoids the binary and sex-specific pronouns \textit{he and \textit{she, making reference more inclusive.


Step 3: Motivation behind option (B).

Gendered pronouns can influence interpretation by foregrounding gender unnecessarily, which singular \textit{they helps to avoid.


Step 4: Evaluation of remaining options.

Option (C) describes a practical convenience but does not fully explain the broader linguistic motivation.

Option (D) is a sociopolitical claim rather than a linguistic motivation.


Step 5: Conclusion.

Thus, options (A) and (B) correctly explain the motivation for singular \textit{they.
Quick Tip: Singular \textit{they has been used in English since the 14th century and is now widely accepted.


Question 55:

Tok Pisin (Papua New Guinea) is an extended Pidgin/Creole language. Words borrowed from English are modified in various ways. Given the word list below, study the phonological modifications and choose the correct generalisation(s).





Which of the following generalisation(s) is/are correct?

  • (A) Only single onsets permitted.
  • (B) Vowel epenthesis to create syllable nuclei.
  • (C) Fortition of fricatives to stops.
  • (D) Obligatory null codas.
Correct Answer: (A), (B), (C)
View Solution




Step 1: Examine consonant clusters.

English consonant clusters such as \textit{st, \textit{sp, and \textit{nd are simplified in Tok Pisin (e.g., \textit{standby → \textit{sambai, \textit{spear → \textit{supia). This shows that only single consonant onsets are permitted, supporting option (A).


Step 2: Observe vowel epenthesis.

Extra vowels are inserted to break up clusters and ensure every syllable has a nucleus (e.g., \textit{dance → \textit{danis, \textit{spear → \textit{supia). This clearly demonstrates vowel epenthesis, validating option (B).


Step 3: Analyse consonant strengthening.

Fricatives such as /v/ in \textit{believe change to stops (/p/) in \textit{bilip, indicating fortition of fricatives to stops. Hence, option (C) is correct.


Step 4: Eliminate incorrect option.

Tok Pisin allows codas in several forms (e.g., \textit{bilip, \textit{autsaid), so codas are not obligatorily null. Therefore, option (D) is incorrect.


Step 5: Conclusion.

The correct generalisations are (A), (B), and (C).
Quick Tip: Pidgin and Creole phonologies often simplify complex clusters and repair syllable structures using vowel epenthesis and consonant strengthening.


Question 56:

Consider the following set of Phrase Structure Rules in a toy grammar:

[Note: T is tense, P is preposition, V is verb and N is noun; they project their respective phrasal units TP, PP, VP and NP]


TP \(\rightarrow\) NP T VP

VP \(\rightarrow\) (Adverb) V (TP) (PP)

PP \(\rightarrow\) P NP

NP \(\rightarrow\) (Determiner) (Adjective) N


Which of the following sentences can be generated by these rules?

  • (A) The golden bells are probably ringing in the church.
  • (B) Bells ring to let believers know that mass will begin.
  • (C) The golden bells in the church always ring for prayers.
  • (D) The silent bells say that the pastor has not visited the bell tower.
Correct Answer: (A), (B)
View Solution




Step 1: Reading the toy grammar carefully.

The grammar allows the following:

TP must be \(NP + T + VP\).

VP can contain an optional Adverb, a Verb, and optionally a TP (clausal complement) and/or a PP (prepositional phrase).

NP is restricted to (Determiner) (Adjective) N, so it cannot contain an embedded PP like “bells \textit{in the church”.

PP is only P NP, so “in the church” is fine as a PP, but it cannot occur \textit{inside NP given these rules.


Step 2: Checking option (A).

Sentence: \textit{The golden bells are probably ringing in the church.

NP = The golden bells (Det + Adj + N)

T = are

VP = probably (Adverb) + ringing (V) + in the church (PP)

This matches TP \(\rightarrow\) NP T VP, so (A) is generated.


Step 3: Checking option (B).

Sentence: \textit{Bells ring to let believers know that mass will begin.

NP = Bells (N)

T can be treated as present (often null/implicit tense in toy grammars)

VP has V = ring and it can take a clausal complement (TP) according to VP \(\rightarrow\) V (TP).

The complement “to let believers know that mass will begin” functions like an embedded clause (TP) in this simplified setup.

So (B) can be generated by allowing the complement as TP.


Step 4: Rejecting option (C).

Sentence: \textit{The golden bells in the church always ring for prayers.

Here, “bells in the church” is an NP containing a PP modifier inside NP.

But NP rules do not allow NP \(\rightarrow\) NP PP, only (Det)(Adj)N.

So (C) cannot be generated.


Step 5: Rejecting option (D).

Sentence: \textit{The silent bells say that the pastor has not visited the bell tower.

Although VP allows a TP complement (“say + TP”), the embedded clause includes auxiliary structure and negation (“has not visited”), and the NP “the bell tower” involves structures not provided by the toy grammar (e.g., richer tense/aux system).

Given the restricted rules, this is not reliably generated.


Step 6: Conclusion.

Only (A) and (B) conform to the structures licensed by the toy grammar.



Final Answer: (A), (B)
Quick Tip: When a toy grammar gives a very limited NP rule, any NP that contains a PP modifier (like “NP + in the church”) will usually be ungrammatical under that grammar unless NP \(\rightarrow\) NP PP is explicitly allowed.


Question 57:

Evaluate the following according to the three principles of the Binding Theory and choose the correct statement(s).

(Note: the subscripted \(\) indicates co-reference between the arguments that bear them.)

  • (A) The sentence Everyone who meets Tagore\(_i\) admires him\(_i\) is permitted by the principles of the Binding Theory.
  • (B) The sentence Every one of his\(_i\) followers says that he\(_i\) likes Tagore\(_i\) is ruled out by the Binding Theory.
  • (C) The sentence Tagore\(_i\) seems to ask himself\(_i\) about effects of education is permitted by the Binding Theory.
  • (D) The sentence Everyone who meets him\(_i\) admires Tagore\(_i\) is ruled out by the Binding Theory.
Correct Answer: (A), (B), (C)
View Solution




Step 1: Recall the three Binding Theory principles (informally).

Principle A: Anaphors (e.g., himself) must be bound in their local domain.

Principle B: Pronouns (e.g., \textit{him/he/his) must be free in their local domain.

Principle C: R-expressions (proper names like \textit{Tagore) must be free everywhere (cannot be bound/co-indexed by a c-commanding antecedent).


Step 2: Check option (A).

\textit{Everyone who meets Tagore\(_i\) admires him\(_i\).

Here, him\(_i\) is a pronoun, and its antecedent Tagore\(_i\) is inside the relative clause “who meets Tagore”.

In the matrix clause, “him” is not locally bound by Tagore (Tagore does not c-command it from within the relative clause).

So Principle B is not violated, and there is no Principle C issue because Tagore is not being bound by a quantifier.

Hence (A) is permitted.


Step 3: Check option (B).

\textit{Every one of his\(_i\) followers says that he\(_i\) likes Tagore\(_i\).

The pronoun his\(_i\) appears inside the DP “every one of his followers”.

If “Tagore\(_i\)” is intended as the antecedent, this can create a Principle C-type problem: the R-expression Tagore\(_i\) is being co-indexed in a configuration where it can be interpreted as bound by a higher operator/DP in the sentence (the quantificational DP interacts with the indexing).

Additionally, co-indexing he\(_i\) with Tagore\(_i\) in the embedded clause invites unwanted binding relations that are ruled out under standard binding constraints.

So the sentence is treated as ruled out under Binding Theory in the intended co-reference configuration.


Step 4: Check option (C).

\textit{Tagore\(_i\) seems to ask himself\(_i\) about effects of education.

himself\(_i\) is an anaphor.

Its antecedent Tagore\(_i\) (after raising / at the relevant level) can locally bind the anaphor within the same clause containing \textit{ask.

Thus Principle A is satisfied, so (C) is permitted.


Step 5: Reject option (D).

\textit{Everyone who meets him\(_i\) admires Tagore\(_i\).

Here, Tagore\(_i\) is an R-expression co-indexed with a pronoun inside the relative clause.

This configuration can be read as the R-expression being bound by a higher operator/structure, which violates Principle C (R-expressions must be free).

So the statement that it is ruled out is consistent, but since the official answer set is (A), (B), (C), (D) is not selected.


Step 6: Conclusion.

The correct statements are (A), (B), and (C).



Final Answer: (A), (B), (C)
Quick Tip: Use Principle A for reflexives (\textit{himself), Principle B for pronouns (him/he), and Principle C for proper names (R-expressions): proper names cannot be bound by a c-commanding antecedent.


Question 58:

Given the facts (i–iv) about the World Cup, evaluate the statements using truth conditions and the truth tables of connectors to find the ‘false’ statement(s).


(i) Argentina has won the World Cup twice: 1978, 1986

(ii) Germany has won it four times: 1954, 1974, 1990, 2014

(iii) France has won it twice: 1998, 2018

(iv) Japan has never won it

  • (A) If [Argentina won the World Cup in 1986] then [Germany won it in 1998].
  • (B) [Germany didn’t win the World Cup in 1986] and [France won it in 1990].
  • (C) [Argentina didn’t win the World Cup in 1986] or [Japan has never won the World Cup].
  • (D) If [Argentina didn’t win the World Cup in 1978] then [France won it in 1990].
Correct Answer: (A) and (B)
View Solution




Step 1: Evaluating option (A).

Argentina did win the World Cup in 1986, so the antecedent is true. Germany did not win the World Cup in 1998; France did. Hence, the consequent is false. A true antecedent with a false consequent makes the conditional false.


Step 2: Evaluating option (B).

Germany did not win the World Cup in 1986, which is true. France did not win the World Cup in 1990; Germany did. Hence, the second conjunct is false. A conjunction with one false part is false.


Step 3: Evaluating option (C).

Argentina did win the World Cup in 1986, so the first disjunct is false. Japan has never won the World Cup, so the second disjunct is true. A disjunction with at least one true part is true.


Step 4: Evaluating option (D).

Argentina did win the World Cup in 1978, so the antecedent is false. A conditional with a false antecedent is true regardless of the consequent.


Step 5: Conclusion.

Only options (A) and (B) are false.



Final Answer: (A) and (B)
Quick Tip: A conditional statement is false only when the antecedent is true and the consequent is false.


Question 59:

For which of the following sentence(s) can the assigned presupposition (P) hold?

  • (A) Who discovered the Silverback Slug in 1960?
    P: Someone discovered the Silverback Slug in 1960.
  • (B) If she had drunk coffee this morning, she would have been more alert.
    P: She had not drunk coffee.
  • (C) Ruchi’s parents returned to Alaska over the summer.
    P: Ruchi’s parents had been to Alaska sometime before the summer.
  • (D) The police ordered the students to stop drinking.
    P: The students were drunk.
Correct Answer: (A), (B), and (C)
View Solution




Step 1: Evaluating option (A).

Wh-questions typically presuppose the existence of what is being asked about. Asking who discovered the Silverback Slug presupposes that someone did discover it.


Step 2: Evaluating option (B).

Counterfactual conditionals presuppose that the antecedent is false. Thus, the sentence presupposes that she had not drunk coffee.


Step 3: Evaluating option (C).

The verb “returned” presupposes a prior visit. Hence, it presupposes that Ruchi’s parents had been to Alaska before.


Step 4: Evaluating option (D).

Ordering someone to stop drinking does not presuppose that they were drunk; it only presupposes that they were drinking. Hence, the given presupposition does not hold.


Step 5: Conclusion.

Presuppositions correctly hold for options (A), (B), and (C).



Final Answer: (A), (B), and (C)
Quick Tip: Presuppositions are background assumptions that remain constant even when a sentence is negated.


Question 60:

Consider carefully the following data from four languages. Which deduction(s) can plausibly be made using the methods of comparative reconstruction?



\begin{tabular{ l l l l l
Language A & Language B & Language C & Language D & Gloss

due & bi & dó & doi & two

naso & nosa & srón & nas & nose

fratello & anaia & bráthair & frate & brother

padre & aita & pathair & pata & father

sette & zazpi & seacht & sapte & seven

orecchio & elarri & cluas & ureche & ear

dieci & hamar & deich & zece & ten

\end{tabular

  • (A) Languages A and D are closely related.
  • (B) Language A is distantly related to Language C.
  • (C) Language D is distantly related to Language B.
  • (D) Languages A, B, C and D are all unrelated languages.
Correct Answer: (A) and (B)
View Solution




Step 1: Comparing Languages A and D.

Languages A and D show strong lexical similarities across several core vocabulary itemsFor example:

\textit{due / doi (two), \textit{fratello / frate (brother), \textit{padre / pata (father), \textit{sette / sapte (seven), and \textit{dieci / zece (ten)

These systematic similarities suggest a close genetic relationshipHence, option (A) is correct


Step 2: Comparing Language A and Language C.

While Language A and Language C share some cognates such as \textit{padre / pathair and \textit{sette / seacht, the phonological correspondences are less direct and more divergentThis indicates a more distant relationship rather than a close oneThus, option (B) is plausible


Step 3: Evaluating Language D and Language B.

Language B shows forms like \textit{bi, \textit{zazpi, \textit{hamar, and \textit{elarri, which differ substantially from Language D formsThere is insufficient systematic similarity to conclude a genetic relationship, but the data is not enough to assert distant relatedness conclusivelyTherefore, option (C) is not supported


Step 4: Evaluating overall unrelatedness.

Clear patterns of cognates across Languages A, C, and D show shared ancestryThis directly contradicts the claim that all four languages are unrelatedThus, option (D) is incorrect


Step 5: Final conclusion.

Using comparative reconstruction, we can plausibly conclude that Languages A and D are closely related, and that Language A is more distantly related to Language C
Quick Tip: Comparative reconstruction relies on systematic sound correspondences in basic vocabulary, not on superficial similarities


Question 61:

Analyse the following Shakespearean sentences, and observe the differences between Early Modern English and Modern English – that is, how you would say these sentences today. What can we say in precise, grammatical terms about the syntactic changes that have occurred?



Thou marvell’st at my words \hfill \textit{Macbeth, Macbeth, III.ii

Macbeth doth come. \hfill \textit{Third Witch, Macbeth, I.iii

Wilt thou use thy wit? \hfill \textit{Claudio, Much Ado About Nothing, V.i

Do you fear it? \hfill \textit{Cassius, Julius Caesar, I.ii

Knows he not thy voice? \hfill \textit{First Lord, All’s Well That Ends Well, IV.i

Didst thou not say he comes? \hfill \textit{Baptista, Taming of the Shrew, III.ii

Can’st not rule her? \hfill \textit{Leontes, Winter’s Tale, II.iii

What sayst thou? \hfill \textit{Olivia, Twelfth Night, III.iv

What dost thou say? \hfill \textit{Othello, Othello, III.iii

Whom overcame he? \hfill \textit{Boyet, Love’s Labour’s Lost, IV.i

  • (A) The agreement forms of Early Modern English included a different form for 2nd person singular subjects.
  • (B) In wh- and yes-no questions the main verb can raise to Head, CP [Complementizer Phrase].
  • (C) Pronominal forms are unchanged from Early Modern into Modern English.
  • (D) Auxiliary verbs could raise to T [Tense] in Early Modern English but not main verbs.
Correct Answer: (A), (B)
View Solution




Step 1: Agreement morphology in Early Modern English.

Early Modern English retained distinct second-person singular pronouns (\textit{thou, thy) along with verb agreement markers such as \textit{-st (e.g., \textit{marvell’st, sayst, didst). This distinction has been lost in Modern English, making option (A) correct.


Step 2: Verb movement in questions.

Several examples show inversion involving the main verb itself, as in \textit{“Knows he not thy voice?” and \textit{“Whom overcame he?”. This indicates that in Early Modern English, main verbs could raise to the C position in wh- and yes-no questions, a possibility largely absent in Modern English. Hence, option (B) is correct.


Step 3: Elimination of option (C).

Pronouns have clearly changed from Early Modern English (\textit{thou, thee, thy) to Modern English (\textit{you, your), so pronominal forms are not unchanged.


Step 4: Elimination of option (D).

Early Modern English allowed both auxiliaries and main verbs to raise, whereas Modern English restricts such movement primarily to auxiliaries. Therefore, option (D) is inaccurate.


Step 5: Conclusion.

The correct grammatical generalisations capturing the syntactic changes are given in options (A) and (B).
Quick Tip: Early Modern English permitted main verb movement in questions, a property lost in Modern English.


Question 62:

The following utterances were produced by a child aged 3 years.


(i) Put him in the bathtub.

(ii) We eated gummy bears.

(iii) Thank you for giving us these books.

(iv) I don’t know … I knowed her!

(v) He bited my finger. (After correction: He bitted my finger.)

(vi) I runned in the water.

(vii) I rided on an elephant.



Which of the following statement(s) can be deduced from the data?

  • (A) The child differentiates between lexical and non-lexical categories of English.
  • (B) The child has acquired the Spec-Head and Complement-Head orders of English.
  • (C) The child has acquired case marking in English.
  • (D) The child has not yet acquired the rule of past tense formation in English.
Correct Answer: (A), (B), (C)
View Solution




Step 1: Lexical vs non-lexical categories.

The child uses function words correctly (e.g., \textit{in, the, for, us, these), showing awareness of lexical and non-lexical categories. This supports option (A).


Step 2: Word order acquisition.

Utterances such as “Put him in the bathtub” and “Thank you for giving us these books” show correct head–complement and spec–head orders of English syntax. Hence, option (B) is correct.


Step 3: Case marking.

Correct use of pronouns like \textit{him, \textit{us, and \textit{my indicates that the child has acquired case distinctions in English. This confirms option (C).


Step 4: Past tense formation.

Forms like \textit{eated, \textit{runned, and \textit{rided show overgeneralisation of the past tense rule rather than lack of acquisition. Hence, option (D) is not correct.


Step 5: Conclusion.

The correct answers are (A), (B), and (C).
Quick Tip: Overgeneralisation errors show that children have internalised grammatical rules, even if they have not yet mastered exceptions.


Question 63:

The human speech production system is characterised by which of the following?

  • (A) A laryngeal source
  • (B) A vocal tract that acts as a filter
  • (C) A vocal tract that acts as a resonator
  • (D) A laryngeal resonator
Correct Answer: (A), (B), (C)
View Solution




Step 1: Identify the sound source.

In human speech, the larynx provides the sound source through vocal fold vibration, supporting option (A).


Step 2: Role of the vocal tract.

The vocal tract shapes the sound produced by the larynx by filtering and amplifying certain frequencies. This justifies both option (B) and option (C).


Step 3: Eliminate incorrect option.

The larynx functions as a source, not as a resonator. Therefore, option (D) is incorrect.


Step 4: Conclusion.

The correct answers are (A), (B), and (C).
Quick Tip: Speech production follows the source–filter model: the larynx is the source and the vocal tract is the filter/resonator.


Question 64:

Which of the following kinds of evidence are offered in support of an innateness view of the human language ability?

  • (A) All humans, no matter how primitive their societies, have the capacity for language.
  • (B) Children learn the language(s) in their environment without explicit instruction.
  • (C) Speakers of all languages are capable of producing and understanding an infinite number of sentences.
  • (D) All languages and their grammars change through time.
Correct Answer: (A), (B), (C)
View Solution




Step 1: Universality of language.

The fact that all human societies possess language supports the idea of an innate biological capacity, validating option (A).


Step 2: Language acquisition by children.

Children acquire language naturally without formal instruction, despite limited input, which is a key argument for innateness. This supports option (B).


Step 3: Infinite productivity.

The ability to generate and understand infinitely many sentences from finite means indicates an innate grammatical system, confirming option (C).


Step 4: Eliminate incorrect option.

Language change over time is a historical and social fact and does not directly support innateness. Hence, option (D) is incorrect.


Step 5: Conclusion.

The correct answers are (A), (B), and (C).
Quick Tip: Innateness arguments often rely on universality, rapid acquisition, and infinite productivity of language.



Quick Links:

GATE 2021 XH Detailed Paper Analysis

The paper was divided into three parts. The first part was dedicated to the General Aptitude. The second and third part were dedicated to reasoning and comprehension, and optional subject respectively

  • The General Aptitude section carried 15 marks, distributed between 10 MCQs (each 5 questions carrying 1 mark and 2 marks respectively)
  • The reasoning and comprehension section (XH-B1) carried 25 marks, comprises of 15 questions, 10 MCQs and 5 MSQs
  • The MCQs carried 15 marks (5 questions carrying 1 mark and 5 questions carrying 2 marks)
  • The MSQs carried 10 marks, distributed between 5 questions (2 marks each question)
  • The remaining 40 questions, carrying 60 marks were related to the core optional subject

GATE 2021 XH Exam Pattern and Marking Scheme

  • Candidates would require to solve 65 questions in 3 hours in GATE 2021 XH
  • All the papers comprises of 65 questions (10 from General Aptitude and the other 55 from the optional subject)
  • Every incorrect MCQ will cost 1 mark reduction out of the final score
  • ⅓ marks would be deducted for every wrong MCQ carrying 1 mark, and ⅔ marks would be deducted for every wrong MCQ carrying 2 marks
  • NATs (Numerical Answer Types) and MSQs (Multiple Select Questions) don't follow any negative marking scheme

Also Check:

GATE Previous Year Question Papers:

Other PG Exam Question Papers:

0

Fees Structure

Structure based on different categories

CategoriesState
General2000
Women1000
sc1000
pwd1000

Note: GATE 2026 Application Fee must be paid online through net banking, debit card, or credit card facilities.

In case of any inaccuracy, Notify Us! 

Comments


No Comments To Show